Главная Юзердоски Каталог Трекер NSFW Настройки

Космос и астрономия

Ответить в тред Ответить в тред
Check this out!
<<
Назад | Вниз | Каталог | Обновить | Автообновление | 579 105 157
Тред Тупых Вопросов №171 171 Edition [ТТВ 171] Аноним 28/12/22 Срд 21:40:09 738638 1
1567514290-171m[...].jpg 250Кб, 1280x1927
1280x1927
000171-asteroid[...].png 18Кб, 250x250
250x250
171OpheliaOrbit.png 9Кб, 821x487
821x487
Kepler-171Orb.png 4Кб, 1592x256
1592x256
Тред вопросов о жизни, Вселенной и всём таком.

Спрашиваем то, за что в других местах выдают путёвку в биореактор. Здесь анонимные учёные мирового уровня критически рассмотрят любые гениальные идеи и нарисованные в Paint схемы.

Q: Можно быстрее?
A: Быстрее до Антарктиды раком.

Q: Почему бы не привязать ракету к воздушному шару или стартовать с горы?
A: Космос - это не как высоко, а как быстро, большая часть энергии ракеты уходит на разгон вбок.
Подробнее тут https://what-if.xkcd.com/58/ (английский) https://chtoes.li/orbital-speed/ (перевод)

Q: В космосе горячо или холодно?
A: В космосе почти вакуум. То разреженное вещество, которое там есть, очень горячее, но на объекты в космосе оно почти не влияет. Влияет излучение. В тени будешь остывать, на солнце нагреваться.

Q: Есть там кто?
A: Не исключено, что нет: https://www.youtube.com/watch?v=PqEmYU8Y_rI
A: Не исключено, что есть: https://www.youtube.com/watch?v=r6rPNPVQp0Y

Q: Я начитался охуительных историй про уфологию, че делать, нам жопа?
A: Да, тебе жопа, можешь сгонять в зогач или куда оттуда пошлют.

Q: Если два корабля летят со скоростью 0.8с навстречу друг другу, то их скорость сближения будет 1.6с АЗАЗАЗЗАЗ Я НАЕБАЛ ИНШТЕЙНА))
A: Не будет, для релятивистских скоростей нужно использовать преобразования Лоренца. Вот хороший цикл коротких видео по СТО, есть англ и ру сабы: https://www.youtube.com/watch?v=1rLWVZVWfdY

Прошлый тред закоррапчен плотноприжатым шизиком: >>737626 (OP)
Аноним 28/12/22 Срд 22:48:40 738644 2
>>738638 (OP)
>Прошлый тред закоррапчен плотноприжатым шизиком
Ну он и этот благополучно засрет, делов-то.
Аноним 28/12/22 Срд 22:50:22 738645 3
Идеальная жидкость польется из жопы плотноприжатого долбоеба через 3, 2, 1...
Аноним 29/12/22 Чтв 09:10:30 738681 4
>>738638 (OP)
Почему про РД-ХУЙ говорят что это всратый принципиально ненадёжный двигатель с задранным давлением, тяжёлый как древнее чугунное говно. А вот НК-33 божественный продукт инженерных гениев, безопасный, с умеренным давлением и лёгкий к тому же, высокий TWR.
Ещё говорят что Глушко был не прозорливым талантливым руководителем, уважаемым, а всего лишь злобным завистливым мудаком который выбрался наверх по головам и погубил советскую космонавтику.
Глушко действительно злобный хуй-мудак, РД-ХУЙ и впрямь перетяжелённое полыхающее дерьмо?
Аноним 29/12/22 Чтв 16:36:26 738702 5
Аноним 29/12/22 Чтв 16:53:45 738705 6
>>738681
Поридж, у тебя женственные манеры и все четыре губы кровоточат уже 7 дней
Аноним 29/12/22 Чтв 17:22:07 738708 7
когда исчезло "кефир вне загона - бан"? там же где навозные жуки?
Аноним 29/12/22 Чтв 18:00:05 738711 8
>>738702
На форумах специалисты из отрасли негативно относятся. А у тебя другие аргументы? Я вот сравниваю характеристики двигателей и вижу что НК-33 лучше во всём, легче в 2 раза и дешевле в разы (оценка стоимости производства времён Бурана), а удельный импульс практически как у РД-ХУЙев всяких. Деятельность Глушко это по факту удушение конкурентов (Кузнецова, Королёва), все их результаты были погублены когда он пришёл к власти. А потом он 10 лет топтался на месте чтобы сделать то же самое (такой же двигатель но хуже и такую же по грузоподъёмности ракету но на 10 лет позже и дороже). Глушко свои личные амбиции поставил выше прогресса государства, так получается. Он и есть злобный мудак получается, губитель всего и достоин только порицания
Аноним 29/12/22 Чтв 19:29:45 738714 9
>>738711
Я уточнил, чтобы убедиться, что вопрос корректный. По идее, у форумных специалистов и надо спрашивать. Почему Глушко посрался с Королёвым, никто не знает и уже не узнает.

С другой стороны, антарес летал на НК-33 и РД-ХУЙ, количество уёбанных Антаресов у НК-33 — один, у рд-хуя — ноль, при том что с рд-хуем он летал намноооого больше. И это при более высоком давлении и УИ. Давление вообще-то больше на 72%, и это охуеть как много. РД-ХУЙ нереально пиздатый двигатель для своего времени (=до появления Раптора)
Аноним 29/12/22 Чтв 19:52:29 738718 10
>>738714
>РД-ХУЙ нереально пиздатый двигатель
Эээ...а с хуя ли он пиздатый если по всем параметрам он хуже НК-33, а по УИ лучше лишь чуть-чуть? И УИ на последнем месте из всех параметров для двигателя 1-й ступени
НК-33 дешёвый в производстве, НК-33 надёжный (прожигался на ресурс несколько тысяч секунд!), НК-33 лёгкий (в 2 раза выше тяговооружённость!), у РД-ХУЙ одни недостатки!
Аноним 29/12/22 Чтв 20:01:49 738719 11
>>738714
>Почему Глушко посрался с Королёвым
Посрался - похуй. Но он отказался делать двигатели для Н-1. То есть взял и затормозил разработку ракеты. Просто так, потому что злобный мудень. Каково, а? Кузнецов с нуля освоил ЖРД, понятно что Глушко бы сделал быстрее (хоть и хуже), а критическую важность времени понимали все!
Аноним 30/12/22 Птн 00:06:38 738726 12
>>738714
>По идее, у форумных специалистов и надо спрашивать. Почему Глушко посрался с Королёвым, никто не знает и уже не узнает.
Масонская хуйня какая-то из серии СССР криптоколония Англии по ДЕГу.
Стратегические КБ это не форум девочек-художниц. Всем похуй, с кем ты там посрался, партия сказала делать - значит делать. И если в уравнении появляется такой откровенный саботаж, после которого никто не расстрелян - значит внутри страны логики нет, ищи бенефициаров снаружи.

Алсо, примерно в то же время поступила указивка прикрыть собственные радиоэлектронные разработки и приступить к копированию западных микросхем. И примерно в то же время СССР начал гнать сибирскую нефть на экспорт, спасая вражеский запад от нефтяного кризиса. Так что саботаж, судя по всему, был системный и согласованный на самом верхнем уровне.
Аноним 30/12/22 Птн 04:37:38 738732 13
image.png 1289Кб, 1280x720
1280x720
image.png 835Кб, 895x572
895x572
image.png 425Кб, 610x369
610x369
Поясните по хардкору нахуя нужна МКС России и в целом мировому сообществу, денег жрет изрядно, а из пользы нашел только смутное "На МКС проводится следующие исследования: медицинские, технологические, биологические, физические, исследования самого космоса и осуществляется мониторинг земли." Исследование космоса и мониторинг Земли можно проводить со специализированных спутников, а остальное это какая-то хуета которую наверняка уже тысячу раз сделали и от безысходности уже таскают на МКС 3д-принтеры и выращивают перчики
Может она является каким-то перевалочным пунктом для подзаправки кораблей при отправлении на Луну? Может там реально прямо сейчас неебаца исследование проводится, которое может помочь человечеству? Или это уже по привычке продляют контракты на поддержку и пилят деньги на этом?
Аноним 30/12/22 Птн 06:39:20 738733 14
>>738726
Стратегическое планирование развития космонавтики со стороны партии в то время отсутствовало как класс.
Была куча КБ которые продвигали свои проекты, пытались получить поддержку и всячески тянули одеяло на себя. Поэтому какое-то работы шли ажно по трем лунным ракетам, Королёвской Н-1, Глушковской УР-700 и Янгелевской Р-56.
Партия же ничего кроме требования постаноки галочек к очереным годовщинам родить не могла.
Аноним 30/12/22 Птн 07:31:17 738734 15
>>738733
>Поэтому какое-то работы шли ажно по трем лунным ракетам, Королёвской Н-1, Глушковской УР-700 и Янгелевской Р-56.
В любом нормальном госпроекте всегда есть несколько штук конкурирующих подпроектов. Ненормально если этого нет. И совсем ненормально если вместо обычной конкуренции идёт взаимный саботаж.
Аноним 30/12/22 Птн 08:36:12 738735 16
>>738734
Глушко саботировал единственный реальный проект лунной ракеты (по остальным ничего и не делалось), он могильщик русской космонавтики, мудак и злодей! Осуждаю Глушко, он должен быть порицаем на всех уровнях, предатель и иуда.
Аноним 30/12/22 Птн 11:26:22 738742 17
Откуда мы знаем[...].webm 1824Кб, 1280x720, 00:00:25
1280x720
Послушайте. Но он же втирает херню.

Ведь в пространстве римана, чем меньше треугольник, тем больше сумма его углов стремится к 180 градусам. Получается, что из такого эксперимента никак нельзя выяснить кривая ли вселенная.
Ведь она может быть сильно больше, чем этот простроенный треугольник, настолько больше, что мы не можем уловить погрешность.
Аноним 30/12/22 Птн 11:31:02 738743 18
>>738742
Вижу ты шаришь, брат! Что думаешь о деяниях Глушко?
Аноним 30/12/22 Птн 11:42:24 738744 19
>>738743
Я не шарю, потому и спрашиваю.
Аноним 30/12/22 Птн 11:48:51 738745 20
>>738726
>партия сказала делать - значит делать
Какой хороший подход. Жаль, не работает. Как и весь калмунизм.

>значит внутри страны логики нет, ищи бенефициаров снаружи
)))
Аноним 30/12/22 Птн 12:05:19 738747 21
>>738745
>калмунизм
чмоня рвется ЛААААЛЛ 🙃🤣👍
Аноним 30/12/22 Птн 17:01:46 738749 22
Напомните пожалуйста, давно где-то читал про какой-то зелёный минерал, который имеет большое отношение к космосу, никак не могу вспомнить название и почему.
Аноним 30/12/22 Птн 19:47:41 738757 23
>>738734
Так не было нормального госпроекта. Не было стратегии. КБ инициативно делали какой-то задел после чего начинали искать в верхах какую-то фракцию которая их поддержит и выбъет ресурсы на дальнейшую реализацию. Нормально было через головы упереть кусок какого-то проекта, как получилось с Салютами, для которых уперли почти готовый военный Алмаз. Наверху понимали практически исключительно "галочки" что "мы первыми сделали" что лучше всего демонстрирует Восход. Тяп-ляп сделаный корабль самоубийц который отлетал джва полета ради пары ачивок.
Аноним 30/12/22 Птн 20:09:50 738758 24
Аноним 31/12/22 Суб 05:26:14 738767 25
>>738749
криптонит, не благодари
Аноним 31/12/22 Суб 11:04:28 738782 26
>>738781
Нет
>Gaia Sky is a libre and open-source astronomy visualisation desktop and VR program with versions for Windows, Linux and macOS
Аноним 31/12/22 Суб 11:47:50 738785 27
>>738767
Нет, ИРЛ минерал, он вроде еще в космосе вокруг планет летает или что-то такое. НУ блин!
Аноним 31/12/22 Суб 11:51:11 738786 28
>>738785
Ну, оливина в метеоритах много, но его и на земле много.
Аноним 31/12/22 Суб 12:04:07 738787 29
2098f3374642bba[...].jpg 10Кб, 300x225
300x225
>>738758
Я бы лучше жил в трехмерной черной вдове.
Аноним 31/12/22 Суб 12:54:14 738789 30
>>738787
у нее сиськи фейк силикон FFF UUUUUUUUUU
Аноним 31/12/22 Суб 14:11:35 738790 31
>>738786
>оливина
Вроде оно.
Аноним 31/12/22 Суб 16:26:10 738792 32
10324698100 40 [...].jpg 427Кб, 1920x1086
1920x1086
Если Земля полая и свод поддерживается внутренним вихрем из плотноприжатых фотонов, то может ли внутри полой Земли существовать жизнь?
Аноним 31/12/22 Суб 16:30:05 738793 33
>>738792
Конечно, там в идеальной жидкости плещутся вероятностные русалки.
Аноним 31/12/22 Суб 16:30:38 738794 34
144982914915338[...].jpg 57Кб, 632x334
632x334
Является ли теорема о расширяющейся Земле подтверждением полой Земли?
Получается, что внутреннее солнце захватывает идеальную жидкость на своих полюсах и порождает массовую материю, которая распирает Землю изнутри.
Тогда гравитация будет только расти, а покинуть Землю будет всё сложнее и сложнее?
Аноним 31/12/22 Суб 16:32:55 738795 35
336.jpg 84Кб, 800x521
800x521
>>738793
По идее там гравитация ниже, значит могли сохраниться динозавры, а поскольку атмосфера более плотная, то они скорее всего ещё и летают.
Аноним 31/12/22 Суб 16:51:58 738798 36
>>738794
Нет вихри их плотноприжатых фотонов будут ускорятся с увиличинием диаметра Земли и центробежная сила будет компенсировать гравитацию. Однако когда диаметр Земли достигнет пределов Роршаха идеальная жидкость прорвет земную кору и планета лопнет.
Аноним 31/12/22 Суб 17:07:25 738800 37
>>738798
>Однако когда диаметр Земли достигнет пределов Роршаха идеальная жидкость прорвет земную кору и планета лопнет.
А где этот предел? Может быть он уже настал и завтра Земля лопнет?
И как будет выглядеть наш лопнувший Мир? Кольцо из осколков? А внутреннее Солнце обрастет новой атмосферой, корой и зародится новая планета?
Аноним 31/12/22 Суб 17:21:21 738802 38
>>738800
>А где этот предел? Может быть он уже настал и завтра Земля лопнет?
Вычисляется он просто: Необходимо радиус Шварцнеггера поделить на постоянную Доска.
>И как будет выглядеть наш лопнувший Мир?
Плотноприжатые фотоны разожмутся и согласно теории Массаракша остатки планеты завернет в пузырь Альбукерке с отрицательной метрикой пространства и время потечет вспять.
Аноним 31/12/22 Суб 17:30:57 738804 39
так алькуберку надувать или заворачивать?
Аноним 31/12/22 Суб 17:35:49 738805 40
>>738804
В положительном пространстве - надувают, в отрицательном - заворачивают, в пространстве нулевого н-типа (не путять с нулевым м-типом) с ним вобще происходят страшные вещи.
Аноним 31/12/22 Суб 17:44:21 738806 41
2-1-1024x1015.jpg 75Кб, 1024x1015
1024x1015
>>738802
Нет, всё же мне кажется, что внутреннее Солнце просто иногда вылетает из оболочки. Марс и Венера - это бывшие оболочки вокруг нашего внутреннего Солнца - поэтому это мертвые планеты без общепланетарного магнитного поля.
Земля живая- у нас есть внутреннее Солнце.
Но возможно однажды внутреннее Солнце покинет и нашу оболочку.
Аноним 01/01/23 Вск 08:18:57 738843 42
>>738785
криптонит существует, просто его ещё не открыли
Аноним 01/01/23 Вск 17:07:16 738856 43
>>738806
Смотрю на картинку и задаюсь вопросом: а Олимп точно вулканическая гора из недр самого Марса? А почему тогда края такие отвесные? Больше похоже, что какой-то крупный многокилометровый астероид упал на поверхность, разрушился под своим весом и от силы импактного удара, в (эпи)центре павшего камня начались бурные реакции и сыгравшие роль имитации вулканической деятельности.
Аноним 01/01/23 Вск 17:13:49 738857 44
>>738856
>А почему тогда края такие отвесные?
Эрозия, беспощадная ты сука.
Аноним 01/01/23 Вск 17:15:04 738858 45
>>738856
> а Олимп точно вулканическая гора из недр самого Марса?
Да.

>Больше похоже, что какой-то крупный многокилометровый астероид упал на поверхность, разрушился под своим весом и от силы импактного удара, в (эпи)центре павшего камня начались бурные реакции и сыгравшие роль имитации вулканической деятельности
А бля, ну раз тебе кажется, что больше похоже, значит всё, ща позвоню в НАСА скажу чтоб везде инфу поправили.
Аноним 01/01/23 Вск 17:42:33 738859 46
>>738856
>Больше похоже, что какой-то крупный многокилометровый астероид упал на поверхность, разрушился под своим весом и от силы импактного удара, в (эпи)центре павшего камня начались бурные реакции и сыгравшие роль имитации вулканической деятельности.
Ты примерно почувствовал?
Аноним 01/01/23 Вск 18:51:00 738863 47
>>738857
>Эрозия, беспощадная ты сука.
Откуда эрозия на Марсе?
Там же если ветерок и дует, то это по факту как домашняя пыль
Аноним 01/01/23 Вск 20:01:41 738868 48
image.png 2935Кб, 1417x1001
1417x1001
>>738857
>Эрозия, беспощадная ты сука.
Очень избирательно беспощадная, надо сказать, Акресус, Павонис и Арсия что-то имеют обычные покатые переходы от горы к местности.
Аноним 01/01/23 Вск 23:03:54 738877 49
А куда делся роскосмос-тред?
Аноним 01/01/23 Вск 23:58:25 738878 50
image.png 47Кб, 224x225
224x225
>>738638 (OP)
Почему ОП-пидорас не объявил о перекате в прошлом треде?
Аноним 02/01/23 Пнд 00:09:12 738879 51
>>738732
Можно устраивать трансляции со святой иконой багамери прямо с орбиты! Все увидят, восхитятся и начнут славить хоспода.
Аноним 02/01/23 Пнд 00:11:16 738880 52
>>738732
И хорошо бы иметь 3d принтер для печати икон! Потому что сегодня багаматерь, а завтра икона святого попыта. Быстренько отпечатал и в продакшен.
Аноним 02/01/23 Пнд 05:47:26 738886 53
>>738863
>Там же если ветерок и дует
За миллиарды лет много надуть можно, плюс огромные перепады температур. Да и когда-то на Марсе вполне себе текли реки и плескались океаны.
>Очень избирательно беспощадная
Как и на земле собственно, иначе все формы рельефа были бы одной и той же формы.
Аноним 02/01/23 Пнд 08:07:18 738891 54
Аноним 02/01/23 Пнд 08:50:03 738893 55
>>738886
>За миллиарды лет много надуть можно
пыль есть пыль - она не приводит к эрозии
>плюс огромные перепады температур
на разных склонах - разный диапазон перепадов
не катит
>Да и когда-то на Марсе вполне себе текли реки и плескались океаны.
ну вот
Аноним 02/01/23 Пнд 08:52:22 738894 56
>>738891
я бы перекатил, но я не слежу за роскосмосокомосом и не в курсе что у них творится
Аноним 02/01/23 Пнд 09:04:02 738895 57
>>738893
>пыль есть пыль - она не приводит к эрозии
По этому можно смело выкидывать воздушные фильтры изо всех устройств
>на разных склонах - разный диапазон перепадов
>не катит
Почему? Где-то у основания днем +20 а ночью - 100, камешек треснул, небольшая осыпь, склон рассыпался враг заходит в город пленных не щадя
Аноним 02/01/23 Пнд 09:17:14 738896 58
>>738895
>По этому можно смело выкидывать воздушные фильтры изо всех устройств
В устройствах фильтры ставятся вовсе не из-за того, что пыль приводит к эрозии
>Почему? Где-то у основания днем +20 а ночью - 100, камешек треснул
Не треснул он - там нет атмосферы, камушек медленно остывает и нагревается излучением
Аноним 02/01/23 Пнд 09:41:31 738897 59
>>738896
На Марсе атмосфера таки есть. Да и какая разница есть или нет, если главное в этом случае быстрые суточные перепады температур.
Аноним 02/01/23 Пнд 10:02:20 738898 60
>>738897
>быстрые суточные перепады температур
чего?
Аноним 02/01/23 Пнд 10:05:18 738899 61
>>738898
Да косноязычно вышло. Большие конечно же.
Аноним 02/01/23 Пнд 11:24:29 738900 62
>>738896
Марс - планета-хуйня, планета-параша, голый бесполезный каменный шар в вакууме, это все знают. На Марсе нету атмосферы, вакуум, нужен скафандр, и так будет всегда потому что атмосферу взять негде, это база. Но этот вакуум на ебаном Марсе, который 600 Паскалей, он влажный. На Марсе высокая влажность, может доходить до 100% и выпадать иней. Поэтому камень может напитаться водой, которая будет жидкая внутри камня, потому что внутри есть соли. И когда ночью начнётся сильный мороз, вода замёрзнет и пойдут трещины. Вот как это работает на бесполезном ваккумном каменном шаре Марсе ебаном. СКАЖИ ТИТАН КРУТО!!!
Аноним 02/01/23 Пнд 12:19:52 738909 63
image.png 64Кб, 222x227
222x227
>>738900
Титан не планета, даже не карликовая планета.
Аноним 02/01/23 Пнд 12:28:54 738911 64
16148216053350.png 527Кб, 900x900
900x900
15963828204380.png 229Кб, 700x531
700x531
>>738909
Спокнись, чмоня. Че там соевые куки на астрономах постановляют меня ащще не ебет. По факту ТИТАН КРУТО и планета возле другой крутой планеты, там будут жить люди. А безатмосферное говно типа меркуризалупы и марсозалупы и нахуй никому не нужно. Я гигашлёпа а ты всего лишь ведомый соевы чмондель, уяснил?
Аноним 02/01/23 Пнд 12:31:37 738913 65
image.png 174Кб, 498x498
498x498
>>738911
Сатурноподсос порвался, несите нового.
Ты понимаешь, что ты даже не у самой крупной планеты обретаешься, газонюх? Полноценные планетобояре доминируют на своих орбитах пока титанонищуки побираются возле больших газовых шаров с десятками других таких же жалких каменюк.
Аноним 02/01/23 Пнд 12:59:57 738914 66
>>738911
На титане распидорасит радиацией.
Аноним 02/01/23 Пнд 14:03:50 738917 67
16718741273110.jpg 20Кб, 128x128
128x128
>>738914
ты дебс неграмотный лох АХВХАХАХВ лалка опущ
Аноним 02/01/23 Пнд 15:42:32 738922 68
Какие условия должны быть на планете(планетоиде) уровня Цереры или Весты, чтобы её можно было колонизировать хотя бы парой поселений и терраформировать в дальнейшей перспективе?
Ну или хотя бы тело размером с Плутон.
Я так понимаю, что КАК МИНИМУМ надо создать атмосферу, приемлемую по давлению, чтобы можно было работать с привычными человеку хоть в чём-то условиями. Ну а учитывая малый вес тела, нужно будет оче много ЦО2 или какого-нибудь аргона, чтобы газ был достаточно тяжёлый, уплотнялся при местной температуре, создавал парниковый эффект и хранил, - насколько возможно - тепло и всё такое прочее.
Аноним 02/01/23 Пнд 19:37:32 738947 69
>>738922
Пока генератор гравитационного поля не завезли - о терраформировании карликовых планет забудь.

Максимум, что там можно сделать - понастроить вокруг планетоида сеть гигантских надувных пузырей по 5-10км в поперечнике, внутри которых уже будут города и биомы. Это не решит вопрос с силой тяжести (спать придётся по-прежнему в центрифугах), но по крайней мере можно будет хоть как-то жить.
Аноним 02/01/23 Пнд 20:40:41 738958 70
>>738922
Чел, НИКАК и НИКОГДА не удастся создать атмосферу на безатмосферном теле. Нужны такие объёмы энергии что построить эти десятки тысяч реакторов (и обеспечивать их "едой") просто невозможно. Поэтому что? Поэтому придётся жить там где атмосфера уже есть. Ещё нужна твёрдая поверхность. Кроме Земли есть только одно такое место - Титан. Да, он холодный и ресурсов там мало, зато можно строить огромные ангары прямоугольных форм с минимумом материалов, теплоизолировать их местным органическим песком (буквально лопатами насыпать в двойную стенку) и жить. Хочешь прогуляться? Одевай штаны и куртку со специальной высокоэффективной теплоизоляцией, глубокий капюшон и кислородную маску. На поясе баллончик с жидким кислородом, хватит на весь день. Вот как-то так, все остальные варианты это скафандр-вакуум-смерть или дурка от постоянной жизни в тесноте
Аноним 02/01/23 Пнд 20:42:39 738959 71
>>738947
>гигантских надувных пузырей по 5-10км в поперечнике
бля чел, эту шизу читать не смешно даже. ты похоже совсем дурачок гуманитарный
Аноним 03/01/23 Втр 02:28:22 738981 72
https://m.youtube.com/watch?time_continue=605&v=oWqdcJs7j7E&feature=emb_title


Так анон, поясни, с чего вдруг длина волны больше что и создаёт красное смещение? Почему она не остаётся прежней, просто волн больше до объекта.


Ведь если взять два разноудаленных объекта, у них же длина волны одинаковые, просто у далёкого их больше.


А красное смещение у нас на стыке видимой вселенной.

Тогда получается что другие объекты, что так же от нас отдаляются по логике видео, так же должны иметь красное смещение, но оно только на краю вселенной.
Аноним 03/01/23 Втр 02:36:20 738982 73
>>738958
Титан говно
Венера круче
Щиты строй
И не пизди
Мудак
вонючий.
Аноним 03/01/23 Втр 02:46:09 738983 74
>>738911
Титан это кусок замёрзшего говна с лужами из метановой мочи. Только сказочный ебанат туда полетит жить поливаемый живительным лучами зашквар от радиационных поясов Юпитера. Другое дело Венера - абсолютно полноценная планета вблизи солнца с атмосферой и гравитацией. Да там немного теплее чем надо и давящая нездоровая атмосфера, но все эти вопросы легко решаемы. Так что у человечества нет другого такого же кандидата на полноценную вторую поланету для жизни. Да, можно немного побаловаться с луной и Марсом, но это бесперспективные направления. Все равно в итоге надо заниматься Венерой, второй планетой, которую люди легко терраформируют и заселят. К тому же и лететь до нее проще простого, она ближе всего.
Аноним 03/01/23 Втр 02:52:50 738984 75
>>738922
Это все уже есть на Венере, надо только от нее часть излучения отвести, цо2 там уже дохуя, в этом и проблема. Поэтому надо отвести излучение от солнца, она начнет охлаждаться, скорее всего весьма быстро, потом углекислота просто выпадет на поверхность в виде снега и все, дальше можно будет уже его хранить, перерабатывать в кислород, воду и обратно нагревать до приятных температур. В идеале ещё магнитосферу создать, но это уже сложнее. Спутник нужен.
Аноним 03/01/23 Втр 11:08:16 738996 76
>>738983
>которую люди легко терраформируют и заселят.
толсто
>К тому же и лететь до нее проще простого, она ближе всего.
толсто
Аноним 03/01/23 Втр 11:10:26 738997 77
не пиздите, на марсе можно догнать атмосферу до 20-30% от земной. нужно скинуть на марс десяток астероидов и дело в шляпе. технология отклонения астероидов уже есть и испытана
Аноним 03/01/23 Втр 11:15:05 738998 78
Ебать тут троль жирнит)) Ну ок, продолжай стараться лол у тибя обязательно получитса
Аноним 03/01/23 Втр 16:01:46 739009 79
>>738984
>Поэтому надо отвести излучение от солнца, она начнет охлаждаться, скорее всего весьма быстро, потом углекислота просто выпадет на поверхность в виде снега и все, дальше можно будет уже его хранить, перерабатывать в кислород, воду и обратно нагревать до приятных температур. В идеале ещё магнитосферу создать, но это уже сложнее. Спутник нужен.
Всего-то и нужно: подгоняем Меркурий к Венере на правах луны, и кидаем на Венеру пару астеройдов изо льда
Аноним 03/01/23 Втр 18:48:01 739015 80
>>739009
Ну это рил сложно, много энергии надо. Но все ещё лучше дристана с абсолютным нулем. Хотя если всякие оумурмура и Борисова часто будут летать, вполне реально их отклонять так чтобы они шваркали об Меркурий в нужном месте. В любом случае это годы.
Аноним 03/01/23 Втр 18:53:39 739016 81
>>738997
Чтобы давление на Марсе было норм надо копать карьер 30км глубиной, тогда там на дне будет норм давление. Но вы ебало околодонных марсиан имаджинируете? Таких попусков во всей системе не сыскать, ну кроме дристанцев, которые вышли на поверхность и замёрзли заживо через пару минут.
Аноним 03/01/23 Втр 20:55:28 739017 82
>>739016
>Чтобы давление на Марсе было норм надо копать карьер 30км глубиной, тогда там на дне будет норм давление
Так вот зачем на Марсе выкопали Маринер.
Аноним 03/01/23 Втр 21:09:23 739018 83
>>738959
Блядь, если 10км полиэтиленовый пузырь для тебя шиза (по сути тот же бигелоу модуль, только больше), то ебучее ТЕРРАФОРМИРОВАНИЕ вообще что-то за гранью разума. Ты хотя бы сопоставляешь сложности задачи?
Аноним 03/01/23 Втр 21:12:27 739019 84
>>738981
>Так анон, поясни, с чего вдруг длина волны больше
Длина волны фотона растягивается вместе с пространством.
Аноним 03/01/23 Втр 22:14:42 739023 85
>>739009
>астеройдов
Как нелепо выглядйт когда й краткое йспользуют вместо й.
Аноним 03/01/23 Втр 22:15:57 739024 86
>>739016
Годная затея, там еще и тепло должно быть. Илон, даешь мохолы на Марсе!
Аноним 03/01/23 Втр 23:37:09 739026 87
>>739019
Пространство не растягивается. В примере объект двигается.
Аноним 03/01/23 Втр 23:45:53 739027 88
>>739026
а, тогда вообрази что волны имеют корпускулярную природу, кек
> Почему она не остаётся прежней,просто волн больше до объекта
потому что не остаётся, большее число волн той же частоты - больше энергии с нихуя
Аноним 04/01/23 Срд 00:46:24 739031 89
>>739026
Если вопрос про космологическое красное смещение, то здесь дело именно в расширении пространства, из-за которого растягивается и длина волны летящего в нем фотона.
Аноним 04/01/23 Срд 01:27:35 739032 90
Откуда на планете взялся ПЕСОК? Если ещё с песком на берегу ручьёв, рек, морей и океанов, то откуда берутся пустыни, которые бывают намного выше уровня моря?
Аноним 04/01/23 Срд 01:36:36 739033 91
>>739032
значительная фракция эрозии любой кучи камней, в чем недоумение то?
Аноним 04/01/23 Срд 02:05:41 739034 92
>>739027
Нихуя не понятно. Ведь вайфай волны так не тянутся.
Аноним 04/01/23 Срд 02:06:24 739035 93
>>739031
То есть в видео >>738981 неверно объяснили.
Аноним 04/01/23 Срд 02:08:31 739036 94
>>739031
> космологическое смещение
Ну тогда получается мы натягиваем невозможность объяснить, на возможные смещения, как ошибки в расчётах на недоказанную тёмную материю.

Тогда вот такие шизы в чем-то правы https://blog-astrofizika.blogspot.com/2018/08/galaxies-redshift.html?m=1
Аноним 04/01/23 Срд 02:12:32 739037 95
Я тут подумал: если у нас есть лес, обычный лес с обычными деревьями, о при каком-нибудь катаклизме могут ли деревья пропитаться каменной пылью или солью настолько, чтобы дерево само по себе пропиталось и окаменело?
Если произойдёт на какой-нибудь планете с лесом катаклизм, из-за которого грунтовые соки с солями пройдут через древесину (например, от продолжающегося и неостанавливающегося нагрева приповерхностной почвы из-за каких-то глубинных нагревов), и затем ещё, например, на поверхности вулканическое облако осело и стало корочкой на коре деревьев такого леса может, по-идее, законсервировать леса на века и тысячелетия, а значит, обнаружь человек такой каменный лес на пустой планете, мы точно сможем знать что вот тут жизнь была, но потом была катастрофа.
Но насколько реален сценарий окаменения леса каким-то таким способом?
Аноним 04/01/23 Срд 02:17:27 739038 96
>>739032
Нет никакого песка, есть мелкие камешки всяких гранитов, полевых шпатов, железистых руд и прочего. Это просто разъебаная и окатаная порода. Когда ледники ехали все это дело чесалось и перемалывалось в песок. Да и приливы отливы и прочие течения, реки за сотни миллионов лет намоют тебе огромное количество песка. И твои пустыни когда то были океаническим дном. Просто условия такие, что чернозем выветривается и остаётся только песок, вот тебе и пустыня.
Аноним 04/01/23 Срд 02:22:36 739039 97
>>739037
При сильном испарении в замкнутом водоеме с океанической водой возможно, что просолятся. Но останутся другие воздействия, которые все равно разрушат. А чтобы окаменело это же должно быть что-то типа стерильной безвоздушной среды, когда завалило чем-то, упало в болота миллионы лет назад. Так что скорее нет, вряд ли возможно.
Аноним 04/01/23 Срд 02:23:20 739040 98
>>739034
тянутся, в 160Мгц вайфае длина волны 2 метра, так что имея на руках нужный осциллограф можно заметить сдвиг даже пешком
вообще все волны тянутся
https://ru.wikipedia.org/wiki/Эффект_Доплера
Аноним 04/01/23 Срд 05:10:46 739042 99
>>739037
>Но насколько реален сценарий окаменения леса каким-то таким способом?
Вполне реален, такие леса существуют. Ну как леса - кучка окаменевших пней или веток.
Аноним 04/01/23 Срд 08:20:56 739044 100
>>739040
>в 160Мгц вайфае длина волны 2 метра
Что за дичь я только что прочитал?
У вайфая всегда была частота 2.4 ГГц, потом ещё 5 ГГц стали появляться. А 160 МГц у вайфая это только ширина спектра сигнала может быть, да и то зависит от стандарта и настроек.
Аноним 04/01/23 Срд 13:17:08 739049 101
>>739040
Волна 5ггц меньше, но чаще 2.4. Отсюда скорость выше и расстояние меньше.
Аноним 04/01/23 Срд 15:03:10 739054 102
>>739036
Чего, блядь? Что за ебаную шизофазию ты высрал? При чем тут темное вещество вообще? Какая невозможность объяснить? Что объяснить? Космологическое красное смещение я тебе только что объяснил. Кто кого куда натягивает? Что ты вообще несешь? Бояры на Новый год перебрал?
Аноним 04/01/23 Срд 16:01:04 739057 103
Ребят, продолжайте про наши ракеты пжлст.
Вот вопрос

Насколько решение в огромном количестве двигателей на лунной ракете Н1 оправдано?

Правда ли, что на протоне можно было устроить пилотируемый облёт Луны?


>>739036
Интересная статья
>>739032
Пепел кремниевой жизни
Аноним 04/01/23 Срд 19:57:55 739076 104
если всё живое на Земле умрет мгновенно, то через сколько лет разрушатся все города, здания, взлетные полосы, техника и тд? Чтобы прям в пыль превратилось. За 1 млрд лет например есть шанс что хоть что то уцелеет от нашей цивилизации? Или там даже самый прочный бетон станет пылью,а на месте городов будут поля и степи, а атомные лодки растворятся в океане?
Аноним 04/01/23 Срд 20:07:24 739078 105
>>739057
>Насколько решение в огромном количестве двигателей на лунной ракете Н1 оправдано?
Само по себе большое количество двигателей могло бы быть оправдано, но Н-1 была по сути одним гигантским клубком взаимосвязанных проблем, и в этих условиях огромное количество двигателей не оправдалось. И да, переход на НК-33 решил бы примерно одну из них, так что доводка ракеты все равно продолжила бы быть адом.
>Правда ли, что на протоне можно было устроить пилотируемый облёт Луны?
Да. В беспилотном варианте корабль даже облетал несколько раз. Но надежность была совершенно недостаточная для пилотируемых полетов, из 14 только 3 более-менее успешно вернулись на Землю, и только один (Зонд-7) сделал это безопасно для возможного экипажа, без спуска по баллистической траектории с ебическими перегрузками.
Аноним 04/01/23 Срд 21:25:08 739086 106
>>739054
А вот и школьник, который не в курсе, что материя именно из-за расхождения придумана.
Аноним 04/01/23 Срд 21:32:03 739087 107
>>739086
Какого расхождения, шиз? Ты отдаешь себе отчет в шизофазии, которую высираешь?
Аноним 04/01/23 Срд 22:51:25 739092 108
1672861884828.png 43Кб, 796x471
796x471
>>739087
> Какого расхождения, шиз? Ты отдаешь себе отчет в шизофазии, которую высираешь?
Аноним 04/01/23 Срд 23:29:04 739095 109
Аноним 05/01/23 Чтв 08:36:56 739109 110
Риши Сунак пообещал сделать математику обязательной для школьников, не достигших 18 лет. Об этом премьер-министр Великобритании публично заявит позже сегодня в своей первой речи в 2023 году.

Глава Великобритании заявил о том, что «необходимо переосмыслить подход к математике» в стране, поскольку большинство школьников не обладают базовыми знаниями по предмету. Планы образовательной реформы будут озвучены уже в ближайшее время, частично — во время первой речи в 2023 году.

«Сейчас лишь половина 16-летних подростков изучают математику. Однако в мире, где повсюду статистические данные, необходимые почти для каждой работы, нашим детям потребуется больше аналитических навыков, чем это было раньше», — отметил Сунак. Премьер-министр также добавил, что данная проблема является для него очень «личной», поскольку он пришел в политику в первую очередь для того, чтобы суметь «дать каждому ребенку самый высокий из всех возможных стандартов образования».



Парадоксальная ситуация сейчас в мире - богатые страны стараются не тратить деньги на образование. Проще переманивать специалистов, а когда больше некого будет переманивать вся цивилизация схлопнется благодаря такому рыночку.
Полная деградация и гниение.
Аноним 05/01/23 Чтв 09:55:25 739110 111
>>739109
И в чем они не правы, если всякие диктаторы типа пыни ебут свой народ в жопу?
Аноним 05/01/23 Чтв 10:19:08 739115 112
>>738997
> не пиздите, на марсе можно догнать атмосферу до 20-30% от земной. нужно скинуть на марс десяток астероидов и дело в шляпе. технология отклонения астероидов уже есть и испытана
и сколько будем делать это?

пару сотен тысяч лет хватит?

чтобы до пенсии успели.
Аноним 05/01/23 Чтв 10:55:12 739118 113
>>739115
Это тролек сальный всего лишь, не обращай внимания, рудольф
Аноним 05/01/23 Чтв 11:02:22 739119 114
>>739095
> джинглбелз
Порвался
Аноним 05/01/23 Чтв 13:36:15 739126 115
>>738984
>она начнет охлаждаться, скорее всего весьма быстро, потом углекислота просто выпадет на поверхность в виде снега и все
Газ то остывает весьма быстро, а миллионы кубических километров горной породы, разогретой до 500К?
Аноним 05/01/23 Чтв 13:39:55 739127 116
image.png 1797Кб, 1600x1600
1600x1600
>>738922
>Я так понимаю, что КАК МИНИМУМ надо создать атмосферу, приемлемую по давлению, чтобы можно было работать с привычными человеку хоть в чём-то условиями.
Кроме как жить под поверхностью вариантов нет. Это и упрощение герметизации, и защита от микрометеоритов и излучения, и теплоизоляция.
Аноним 05/01/23 Чтв 13:52:34 739129 117
>>739118
То есть по факту аргументов у тебя нет? Алсо 2 астероида около марса уже есть, нужно притащить ещё немного.
Аноним 05/01/23 Чтв 13:56:05 739131 118
>>739129
>Алсо 2 астероида около марса уже есть, нужно притащить ещё немного
Один из них сам скоро упадет уже, надо только немного подождать.
Аноним 05/01/23 Чтв 14:31:03 739132 119
>>739126
Тоже остынут, отдавая тепло сначала газу, потом снегу углекислотному. На земле дельта день ночь около десяти градусов. В пустыне ещё хуже несмотря на то что песок раскален. Так что и на Венере породы остынут весьма быстро. Самый пиздец потом этот снег где-то хранить, чтобы он опять не поплавился. Возможно поначалу надо будет просто отгрести говны от экватора ближе к полюсам и прогревать только экватор. И так постепенно расширяя полосу экватора, насколько позволят мощности по переработки снега в кислород и воду. Ещё она небось плоская вся и надо будет каналы рыть чтобы вода туда сливалась, иначе все это перемешается и получится водоуглекислтный дрист. Короче да, так подумать, работы там пиздец. Но оно того стоит конечно.
Аноним 05/01/23 Чтв 15:21:16 739139 120
>>739132
>На земле дельта день ночь около десяти градусов
Это температура воздуха, а на метровой глубине даже летом холодно.
>Самый пиздец потом этот снег где-то хранить, чтобы он опять не поплавился
Чтобы хранить углекислый газ в твердом виде надо чтоб температура нигде и никогда не поднималась выше чем рекорды отрицательных температур на земле, а оно нам не надо.
Аноним 05/01/23 Чтв 17:10:50 739149 121
>>739078
Благодарю за ответы

> Но надежность была совершенно недостаточная для пилотируемых полетов

Почему? Торможение со второй космической никогда не практиковалось с людьми на борту?
Аноним 05/01/23 Чтв 19:30:59 739158 122
>>739149
>Почему?
Аварии происходили постоянно на всех этапах, вот почему. Протон падал через раз в те времена. Блок Д который должен был разгонять корабль к Луне отказывал. Облетели луну 4 корабля из 14 (два из них запускались на Н-1) и 3 из них столкнулись с серьезными проблемами. У одного (зонд-6) по не до конца установленным причинам (грешили на проблемы терморегуляции и разгерметизацию) отказала парашютная система и корабль разбился. У двух (зонд-5 и зонд-8) из-за проблем с системой ориентации был баллистический спуск вместо аэродинамического и ебические перегрузки. Полностью программу полета выполнил только один корабль, Зонд-7.
>Торможение со второй космической никогда не практиковалось с людьми на борту?
Практиковалось на Аполлонах. СССР свою лунную программу до пилотируемых полетов не довел. Но проблема Зондов была не в самом торможении со второй космической, Зонд-7 его провел безопасно для человека, а в постоянных срывах в баллистический спуск, когда корабль быстро зарывался в плотные слои атмосферы вместо того что бы используя аэродинамическое качество плавно сбрасывать скорость в верхних слоях. Срывы вызывались отказами системы ориентации, из-за которых не удавалось произвести коррекции траектории, необходимые для входа в атмосферу под нужным углом.
Аноним 06/01/23 Птн 09:57:03 739216 123
image.png 208Кб, 500x333
500x333
Если взрывать метеориты одного размера (ладно, очень мощне бомбуэ) на одно и то же место, то что с кратером будет? Стенки при первом ударе образуются, а потом они станут выше относительно дна кратера или наоборот их не станет вовсе?
Образуется ли выпуклость по центру при взрыве бомбуэ или это только при метеорите?
Аноним 06/01/23 Птн 10:38:31 739227 124
>>739216
Кратер будет углубляться но постепенно его начнет засыпать выбросами и глубина стабилизируется.
Аноним 06/01/23 Птн 14:15:49 739241 125
>Считается, что 300—500 млн лет назад Венера претерпела событие, которое привело к полному обновлению коры планеты или к перекрытию её верхних слоев поступившим мантийным материалом. Одним из возможных объяснений этого явления является гипотеза о цикличности подобных событий, в результате которых происходит избавление от излишков тепла, накапливаемых в её внутренних слоях в течение продолжительного времени
Почему считается что температура Венеры это следствие парникового эффекта, а не результат нагрева изнутри?
Аноним 06/01/23 Птн 14:22:39 739242 126
>>739241
так у земли же нет нагрева изнутри
Аноним 06/01/23 Птн 16:17:26 739248 127
>>739241
Суть парникового эффекта не в источнике тепла, а в том что его избыток не стравливается наружу.
Аноним 06/01/23 Птн 17:04:53 739251 128
>>739242
>так у земли же нет нагрева изнутри
Шта
Аноним 06/01/23 Птн 17:06:31 739252 129
c6Eg9c3sJfSRA-I.mp4 5350Кб, 1280x720, 00:00:58
1280x720
Почему середина поднимается медленнее всего?
Аноним 06/01/23 Птн 21:21:50 739267 130
1673029237565.jpg 28Кб, 660x482
660x482
Спутники не могут удерживаться на орбитах спутников планет? Например у спутников Юпитера.
Аноним 06/01/23 Птн 21:45:01 739269 131
Аноним 06/01/23 Птн 22:21:36 739276 132
Насколько осуществима идея запустить в атмосферы газовых гигантов дирижабли? Они в принципе могут, наверно, довольно долго болтаться в верхних слоях, и разбиться им не обо что.
Аноним 07/01/23 Суб 00:08:16 739286 133
>>739267
>Спутники не могут удерживаться на орбитах спутников планет?
Могут но только внутри сферы хилла. Если спутник движется на границе, то орбита слишком возмущенная и неустойчивая.
>Например у спутников Юпитера.
За исключением Ио, у них сфера хилла чуть меньше тыщ км. Так что вполне есть годные орбиты.
Аноним 07/01/23 Суб 00:11:24 739287 134
>>739267
Могут. У спутников есть сфера Хилла как и у планет. Эмпирическое правило таково, что внутри сферы Хиллп орбита стабильная если её радиус не более 1/10 радиуса орбиты самого спутника. То есть для Ио это радиус 35 000 км что весьма немало
Аноним 07/01/23 Суб 00:13:45 739288 135
>>739287
Но как заметил анон выше >>739286 если сфера Хилла очень маленькая то это правило не работает. Всё равно спутники спутников могут быть, например камешки в кольцах вокруг Реи и (в прошлом) Япета.
Аноним 07/01/23 Суб 00:17:30 739289 136
>>739276
Атмосфера газовых гигантов сильно отличается от астросферы земли.
В верхних слоях она состоит из горячего разреженного водорода и гелия, которую дают очень маленькую выталкивающую силу. для "плавания" там надо заправляют дирижабли вакуумом.
В средних слоях уже присутствуют "холодный" водород и гелий и тяжелые молекулы вроде аммиака и метана. Но тут опять слишком низкая выталкивающая сила для дирижабля.
В нижних слоях уже сильно растет температура и появляются плотные облака из тяжелых молекул, тут можно "плавать", но тут дуют ебучие ветра на половине скорости звука. Забудь об управлении.
Аноним 07/01/23 Суб 00:40:12 739290 137
Кстати сфера Хилла у Титана и Каллисто не >>739286
>чуть меньше тыщ км
а 50 000 км, а у Ганимеда 30 000 км
Аноним 07/01/23 Суб 00:42:26 739291 138
>>739289
>Забудь об управлении.
Ну, на данном этапе - хуй с ним, с управлением. Хорошо, что можно собирать метеоданные и щупать ГГ в глубину излучением. Я думаю, эта информация всё равно будет довольно ценной.
Аноним 07/01/23 Суб 02:17:38 739292 139
>>739291
Да, никаких проблем с дирижаблем нет. Надо только запустить его в нужную зону - без ураганов. Чтобы дирижабль летал его нужно надуть забортным водородом и подогревать (кислородной горелкой + теплотзоляция)
Аноним 07/01/23 Суб 02:54:04 739293 140
>>739139
Это временная мера на этапе терраформирования. Углекислота должна быть в виде снега и льда, иначе опять привет ебовое давление при котором работы невозможны. Так что остаётся только держать в твердой фазе и постепенно перерабатывать преимущественно в воду. Правда ещё откуда то надо водород брать для этого, возможно как-то из пород извлекать.

Про поверхность - да, она должна остыть, но так то и на земле через несколько километров уже начинает к сотне градусов подбираться температура. Так что на Венере тоже вовсе не нужно ждать когда она вся остынет, достаточно пары километров. Атмосфера видимо будет долго мешать остывать поверхности, отражая тепло, но когда она в основном вымерзнет дело должно веселее пойти.
Аноним 07/01/23 Суб 02:58:11 739294 141
>>739286
>>739287
Понял, принял, поблагодарил за консультацию.
Аноним 07/01/23 Суб 10:41:24 739315 142
>>739293
Проще сдуть атмосферу, чем ждать когда она остынет.
Аноним 07/01/23 Суб 10:52:12 739318 143
>>739291
>Хорошо, что можно собирать метеоданные и щупать ГГ в глубину излучением
У газовых гигантов атмосфера не радипрозрачна.
Аноним 07/01/23 Суб 11:41:38 739325 144
>>739318
кто сказал? ты лох чмо чмоня ВАХПХАХАХА
Аноним 07/01/23 Суб 11:42:35 739326 145
>>739241
Ну так Венера ближе к солнцу и по этой причине получает больше энергии, а атмосфера нагрев ещё более усиливает. Без такой густой атмосферы там на поверхности не было бы 450 градусов.
Аноним 07/01/23 Суб 11:50:53 739329 146
>>739315
И куда ты ее сдуешь и зачем? Атмосфера Венеры это преимущество, а не недостаток, это огромный запас кислорода и уже готовой углекислоты для фотосинтеза. Углерод тоже куда-нибудь пригодиться.
Аноним 07/01/23 Суб 12:11:53 739333 147
>>739329
Солнечным концентратором, который по размерам будет меньше, чем затеняющее зеркало.
По целому ряду причин кислород предпочтительнее добывать из силикатов и прочих твердых оксидов. И кислород и так самый распространенный элемент на Венере, стоит вопрос как от него избавится.
Углерод же можно без терраформации утилизировать.
Аноним 07/01/23 Суб 23:09:16 739405 148
Если мы удалим все планеты из солнечной системы, а потом расположим на равном расстоянии друг от друга копии Земли в водной плоскости по экватору Солнца, начиная с самой близкой к Солнцу по расстоянию точки положения Меркурия на орбите и заканчивая самой дальней от Солнца точкой орбиты Нептуна, то сколько земель может оказаться в солнечной системе, при условии, что земли находятся друг к другу достаточно близко, чтобы при пролётке мимо друг друга теребить гравитацию, магнитосферы и тектонику друг друга в той мере, в какой Луна влияет на Землю, но не более того?
Аноним 07/01/23 Суб 23:25:52 739406 149
>>739405
много, но сами собой они так образоваться не смогут, а значит и рассуждать тут не о чем
Аноним 08/01/23 Вск 00:34:40 739412 150
>>739405
>земли находятся друг к другу достаточно близко, чтобы при пролётке мимо друг друга теребить гравитацию, магнитосферы и тектонику друг друга в той мере, в какой Луна влияет на Землю, но не более того
Импоссибру. При таких близких пролетах они орбиты себе пораспидорашивают и поврезаются/повышвыривают друг друга.
Через сколько-то мегалет останется полдюжины земель самых разных размеров, с лунами и кольцами, и все с полностью распидорашенными поверхностями.
Аноним 08/01/23 Вск 00:38:52 739413 151
>>739412
>Импоссибру. При таких близких пролетах они орбиты себе пораспидорашивают и поврезаются/повышвыривают друг друга.
>Через сколько-то мегалет останется полдюжины земель самых разных размеров, с лунами и кольцами, и все с полностью распидорашенными поверхностями.
А если подкатить Венеру к Земле? Не на орбиту Луны, а просто подвинуть орбиту Венеры поближе к орбите Земли, чтобы Земля своим гравитационным полем теребила грави- и магнитосферу Венеры?
Аноним 08/01/23 Вск 00:43:36 739415 152
image.png 285Кб, 634x650
634x650
>>739413
>магнитосферу Венеры
Не рассказывайте ему...
Аноним 08/01/23 Вск 00:48:44 739417 153
>>739405
Планетой нынче считается тело, способное расчистить от посторонних тел свою орбиту. Так что куча набитых на орбиты земель друг друга распидорасят, пока одна не останется, разве что в точках L3 что-то выживет.
Ну и орбитальные резонансы никто не отменял. Так что будет примерно та же хуйнюшка. Часть планет встанет на место нынешних, часть слипнется в огромные Юпитеры, часть побьётся в астероидный пояс.
Аноним 08/01/23 Вск 01:19:01 739421 154
16731105703330.png 863Кб, 1080x845
1080x845
d7072942edf1ab5[...].png 499Кб, 730x485
730x485
5.-Huygens-land[...].jpg 100Кб, 800x594
800x594
>>739417
>Планетой нынче считается
АХПХАХПХА ну вы посмотрите на этого послушного омежечку ЛОООЛ. Уже прочитал что там дяденьки из IAU решили? Со всем согласен, да? Теперь только так, да? ХАРРРТЬФУ какой же ты сойкукич жалкий даже хуже соевых из IAU которые четотам "решили". А меня блять не ебёт че там порешал жирный пиздабол Michael E. Brown, пусть в жопу себе засунет своё очень важное мнение. Если такие опущи как Марс и Меркурий планеты, то Титан тем более планета. По совокупности признаков. Это понятно? Это понятно.
АНУ ССУКА БЫРО СКАЗАЛ ТИТАН КРУТО ничтожный сойбой!!!
Аноним 08/01/23 Вск 01:55:20 739422 155
image.png 192Кб, 633x356
633x356
>>739421
Джерри, полезай в корабль, хватит о планетах мечтать, куколд обоссанный, моя дочь умудрилась найти хахаля хуже тебя, лучше пусть с тобой будет.
Аноним 08/01/23 Вск 05:39:31 739425 156
>>739076
> если всё живое на Земле умрет мгновенно, то через сколько лет разрушатся все города, здания, взлетные полосы, техника и тд? Чтобы прям в пыль превратилось. За 1 млрд лет например есть шанс что хоть что то уцелеет от нашей цивилизации? Или там даже самый прочный бетон станет пылью,а на месте городов будут поля и степи, а атомные лодки растворятся в океане?
Фильм Life after people. Он же жизнь после людей. Есть на тыртрубе. Там есть все что тебе надо. За достоверность расчётов не ручаюсь.
Аноним 08/01/23 Вск 08:35:19 739429 157
>>739422
>Рик и Чморти
Ля, и вправду соя...
Аноним 08/01/23 Вск 09:49:54 739452 158
Аноним 08/01/23 Вск 12:41:47 739478 159
>>739425
>1 млрд лет
Чел челик челибос ты сам-то подумай! 1 мдрд лет конечно "бетон станет пылью" это даже и не обсуждается. Даже за 1 млн лет станет
>>739429
хехехе а ты думал?
Аноним 08/01/23 Вск 13:05:15 739482 160
>>739478
> >1 млрд лет
> Чел челик челибос ты сам-то подумай! 1 мдрд лет конечно "бетон станет пылью" это даже и не обсуждается.
Мне тут не с чем спорить, кроме того что ты отвечаешь на ответ. Вопрос задавал не я.
Аноним 08/01/23 Вск 13:23:12 739486 161
Насколько слабая атмосфера Плутона?
Если гипотетического чела, не чувствительного к перепадам давления от вакуума до 100 атмосфер, послать гулять с кислородным баллоном по поверхности Плутона в шубе, будет челу норм? Там же, по-идее, атмосфера слабее марсовой, настолько слабая, что даже холода ты не почувствуешь, ведь тебя молекулы холодные даже особо не заденут и тепла не заберут
Аноним 08/01/23 Вск 14:32:32 739492 162
>>739486
>Насколько слабая атмосфера Плутона
OCHE слабая.
>Если гипотетического чела, не чувствительного к перепадам давления от вакуума до 100 атмосфер, послать гулять с кислородным баллоном по поверхности Плутона в шубе, будет челу норм?
Не будет. Надо герметичный шлем, и то может быть из ануса травить будет. Нет.
Капилляры полопаются, вся открытая слизистая выкипит.
>Там же, по-идее, атмосфера слабее марсовой, настолько слабая, что даже холода ты не почувствуешь, ведь тебя молекулы холодные даже особо не заденут и тепла не заберут
Ага, ты перегреешься нахуй и помрешь от гипертермии (гипер - значит много, термос - значит температура, высокая температура)
Там 12 миллибар, блжад, многие компактные вакуумные установки до такого давления сосать не могут, это считай вакуум и есть.
Аноним 08/01/23 Вск 14:58:02 739493 163
>>738638 (OP)
C какой скоростью распространяется температура? Как это посчитать например в воде, воздухе, ртути, гелии?
Аноним 08/01/23 Вск 15:08:56 739494 164
>>739493
Насколько я помню со со школы - зубодробительными формулами теплотехники.
Аноним 08/01/23 Вск 15:10:15 739495 165
>>739494
То есть даже примерно с учётом температуры и давления нельзя прикинуть?
Аноним 08/01/23 Вск 15:13:03 739496 166
И ещё вопрос. Где посмотреть фото в высоком разрешении интерференции? Что нашёл все микроскопические
Аноним 08/01/23 Вск 15:17:16 739497 167
>>739495
Там и время, и разность температур, и теплопроводность, и фаза луны. И все это хитрым образом делится и перемножается друг с другом.
Аноним 08/01/23 Вск 16:07:25 739500 168
>>739493
>Как это посчитать например в воде, воздухе, ртути, гелии?
Промышленными системами моделирования CFD за сотни нефти. Если ты на коленке ради фана решил помоделировать перенос тепла в турбулентном потоке — земля тебе пухом, братишка.
Аноним 08/01/23 Вск 16:11:59 739501 169
>>739500
>>739497

Л-ладно, я всё понял
У меня есть движок годот и идея как это смоделировать!
Аноним 08/01/23 Вск 16:50:17 739503 170
Правильно ли я думаю, что если у нас есть малюсенькая чёрная дыра, то, в принципе, человек сможет построить вокруг чёрной дыры что-то типа сферы Дайсона, только не для черпания энергии, а для того, чтобы надстроить на ней искуственную поверхность с землеподобной гравитацией на поверхности сферы?
То есть, в таком случае, если горизонт событий будет не больше размера какой-нибудь планетоиды, типа Плутона, то особо-то проблем для создания искусственной планеты вокруг чёрной дыры с любой удобной гравитацией (по нуждам, в зависимости от высоты над горизонтом событий) быть не должно? И тогда на таком объекте можно даже биосферу создать, если площадь поверхности позволяет
Аноним 08/01/23 Вск 17:15:29 739505 171
Можно ли сделать хороший телескоп и высадить его на Хароне. Так мы получим самую далекую точку обзора вселенной? Там же как и на Плутоне атмосферы практически нет, геологическая активность низкая. И пыли нет, только амичано-водяной снег, да он еще и испаряется. Единственное солнца нет для батареи, но можно же ядерную батарею присоединить. Ладно, почему всякие планетоходы забрасывают только на Марс и Луну?
Аноним 08/01/23 Вск 17:22:56 739506 172
>>739505
Для начала было бы интересно такой посадить на Луну

Из недостатков проблемы со связью

Видимо без базы ТАМ никак, верно и для Харона
Аноним 08/01/23 Вск 17:37:14 739509 173
image.png 548Кб, 1200x991
1200x991
>>739503
Номаи в треде, все в спасательную капсулу!
Аноним 08/01/23 Вск 17:49:37 739510 174
>>739505
>Так мы получим самую далекую точку обзора вселенной?
Ага, и увидим с этой точки абсолютно такую же вселенную, какую видим с Земли, не считая милипиздрического смещения. Задачи у этого телескопа какие?
Аноним 08/01/23 Вск 18:36:27 739513 175
>>739510
Рассматривать солнечную систему изнутри, следить за облаком Оорта, спутник без атмосферы, без пыли в отличие от Луны.
Аноним 08/01/23 Вск 19:34:57 739516 176
1s.png 408Кб, 705x387
705x387
Cкок ещё проживёт человечество?
Аноним 08/01/23 Вск 19:54:45 739521 177
>>739516
7 лет 3 месяца и 15 дней. Можешь скринить.
Аноним 08/01/23 Вск 21:25:38 739528 178
>>739516
На Земле уже было как минимум 5 массовых вымираний. Ледниковый период вряд ли нас истребит, но очень сильно отбросит в развитии. Защиты от астероидов у нас нет, так что любой большой астероид сотрёт нас с лица Земли. Ядерная война нас не уничтожит, максимум отбросит в развитии лет на 30.
Аноним 08/01/23 Вск 22:38:42 739540 179
>>739493
>C какой скоростью распространяется температура?
Так температура - это просто скорость, с которой летают молекулы воздуха. Могут иметь скорость до сотен км/с.
Аноним 08/01/23 Вск 23:27:09 739550 180
Аноны, звёзд мало на небе, я уже неделю вижу максимум 6-7 звёзд, У вас как?
Аноним 09/01/23 Пнд 05:10:52 739580 181
>>739550
У нас перманентное световое загрязнение и никогда больше пары самых ярких звезд и основных планет ничего не было видно. Может когда натриевые на диоды поменяют что-то видно станет, но не зимой, когда альбедо у земли высокое.
Аноним 09/01/23 Пнд 09:06:32 739583 182
>>739580
Не, даже из Москвы больше видно, если не из самого центра и очки или линзы по вкусу ещё надеть. И даже зимой. Да даже с Садового кольца больше можно разглядеть, если в не слишком освещенный двор зайти и немного постараться.

А натриевые лампы (высокого давления) хоть более-менее линейчатый спектр дают, светодиодные то практически сплошной спектр имеют. Лучше всего конечно натриевые лампы низкого давления...
Аноним 09/01/23 Пнд 09:57:22 739585 183
>>739583
Пф, так у москвы не особо сильное световое загрязнение. Большая деревня.
Аноним 09/01/23 Пнд 10:51:15 739589 184
>>739580
так ещё неделю назад видно было нормально звёзды, а щас вобще бывало за ночь одну всего увидишь. может дымка или ещё какая погань в небе. Хотя не яркие звёзды штуки 3 сёдня ночью было
Аноним 09/01/23 Пнд 11:32:09 739593 185
>>739589
после праздников не все включили еще
Аноним 09/01/23 Пнд 17:30:18 739623 186
image.png 306Кб, 640x463
640x463
image.png 2133Кб, 1600x901
1600x901
Аноним 10/01/23 Втр 06:41:16 739665 187
image.png 787Кб, 1200x798
1200x798
>>739589
Забыл где живёшь? Попробуй при таком небе ночью хоть что-то разглядеть.
Аноним 10/01/23 Втр 23:43:38 739735 188
почему Юпитер не стал звездой? Что ему помешало? Массы не хватило?
Аноним 11/01/23 Срд 00:19:21 739737 189
>>739735
>Массы не хватило?
Это.
Скорость света и астрономические наблюдения Аноним 11/01/23 Срд 01:01:41 739738 190
image.png 605Кб, 800x800
800x800
Анонче, есть вопрос. Вот каким образом колдуны-учёные, к примеру, создают подобные визуализации, учитывая то обстоятельство, что звёзды находятся от Земли на разных расстояниях, и свет от них доходит до нас с разной скоростью?
Вот, допустим, есть какая-нибудь звезда на подзалупках нашей галактики. Свет от неё доходит до нас за десятки тысяч лет. За это время сама она уже пролетает очень много километров. Как узнать расстояние до неё?
Аноним 11/01/23 Срд 01:09:54 739739 191
>>739738
Описался: конечно, свет идёт не с разной скоростью, а разное время от звёзд различной удалённости.
Аноним 11/01/23 Срд 01:31:04 739740 192
apj472701f8lr ([...].jpg 59Кб, 600x617
600x617
apj472701f7lr ([...].jpg 62Кб, 600x600
600x600
>>739735
Всего 10-11 юпитеров сверху, и он бы светил 400-1000 млн. лет как дейтериевая звезда. Но 10-11 юпитеров это не "чуть-чуть нехватило", это нехватка довольно таки серьёзная))
К тому же чтобы стать дейтериевой звездой нужно быть определённой массы, диапазон очень узкий (11-13 масс Юпитера). Если больше то будет не дейтериевая звезда а коричневый карлик обычный (лох)
Аноним 11/01/23 Срд 03:39:38 739742 193
>>739738
> Анонче, есть вопрос. Вот каким образом колдуны-учёные, к примеру, создают подобные визуализации,
Моделирование какое-нибудь, картография условная, на ее базе уже что-то рисуют наверное.
> учитывая то обстоятельство, что звёзды находятся от Земли на разных расстояниях, и свет от них доходит до нас с разной скоростью?
> Вот, допустим, есть какая-нибудь звезда на подзалупках нашей галактики.
Угу.
> Свет от неё доходит до нас за десятки тысяч лет.
Угу.
> За это время сама она уже пролетает очень много километров. Как узнать расстояние до неё?
Свечи, смещение. Вроде бы гайя одним своим существованием дала невиданный доселе параллакс, но вряд ли и ее до задворок хватит (могу ошибаться). Определением расстояний астрометрия занимается, тебе туда (куда?а никуда нахой, у нас нет треда по астрометрии)
Аноним 11/01/23 Срд 04:18:24 739743 194
>>739505
> Можно ли сделать хороший телескоп и высадить его на Хароне. Так мы получим самую далекую точку обзора вселенной? Там же как и на Плутоне атмосферы практически нет, геологическая активность низкая. И пыли нет, только амичано-водяной снег, да он еще и испаряется. Единственное солнца нет для батареи, но можно же ядерную батарею присоединить. Ладно, почему всякие планетоходы забрасывают только на Марс и Луну?
Вы находитесь здесь->Орбита-->спускаемый аппарат->планетоход. Спускаемых телескопов еще не делали вообще. Спускаемый аппарат был на Титане. Выбить финансирование под Плутон будет архисложно, так как это десяток миссий, ворох нерешённых проблем, огромные риски провала на каждом этапе. При том, что есть идеи куда более потенциально результативные (те же спутники Сатурна, Юпитера), и т.п. Выбить финансирование на спускаемый к Меркурию (при всей бесполезности этой идеи) проще было бы, чем на орбиту к Плутону.
Аноним 11/01/23 Срд 04:49:10 739745 195
>>739738
> каким образом колдуны-учёные, к примеру, создают подобные визуализации
экстраполируют данные других видимых галактик
> Вот, допустим, есть какая-нибудь звезда на подзалупках нашей галактики
в контексте твоего вопроса вообще поебать на неё, различимы только пару соседних рукавов и уплотнение в центре, остальное по сути художественная подгонка
> Свет от неё доходит до нас за десятки тысяч лет.
> Как узнать расстояние
опять в том же контексте поебать на сдвиг положения относительно наблюдаемого, поскольку оборот вокруг галактики звёзды делают за сотни миллионов, а не десятки тысяч лет
Аноним 11/01/23 Срд 04:57:39 739747 196
>>739743
> на спускаемый к Меркурию (при всей бесполезности этой идеи
странны объект для сарказмирования, челы на серьёзных щщах фантазируют о ещё одном лендере на винес
Аноним 11/01/23 Срд 05:48:36 739748 197
>>739747
> > на спускаемый к Меркурию (при всей бесполезности этой идеи
> странны объект для сарказмирования, челы на серьёзных щщах фантазируют о ещё одном лендере на винес
Не странный. Винес то как раз куда более интересно. И что значит фантазируют. Роскосый всерьёз проект выкатывал.
Аноним 11/01/23 Срд 09:05:22 739751 198
16703854902680.jpg 20Кб, 275x183
275x183
>>739740
а если в качестве розжика садануть в него охуенно пиздатую термоядерную бонбу, загорит?
Аноним 11/01/23 Срд 09:36:14 739753 199
>>739743>>739505
> Спускаемых телескопов еще не делали вообще
Спускаемые телескопы нахуй не нужны.
Если ты можешь поднять большую глазастую ёбу в космос - то выгоднее её в невесомости и оставить, где избыточные конструкции не нужны и обзор всегда на 360 градусов. Ещё и всякие бонусные штуки типа Starshade можно закинуть.

>>739513
>Рассматривать солнечную систему изнутри
Это желательно делать как можно ближе к Солнцу, чтобы оно рассматривать не мешало, а наоборот подсвечивало.
>следить за облаком Оорта
Облако Оорта простирается так далеко, что в принципе похуй что с Харона, что с Меркурия его рассматривать.
>спутник без атмосферы, без пыли в отличие от Луны.
На орбите ни того, ни другого нет.
Аноним 11/01/23 Срд 10:50:03 739759 200
>>739751
Нет. Митивариты всякие, всякие кометы Шумахера-Леви несут в себе больше мощей чем любой ядрен-батон который ты мог бы сбросить. Если бы можно было поджечь юпитер, он бы уже сгорел.
Аноним 11/01/23 Срд 12:06:51 739762 201
>>739521
думаешь людишки дождутся когда будет красивая дата и выпилятся нахуй?
Аноним 11/01/23 Срд 12:07:23 739763 202
>>739521
думаешь людишки дождутся когда будет красивая дата и выпилятся нахуй?
Аноним 11/01/23 Срд 12:09:09 739764 203
>>739759
но они ведь не взрываются с огнём, надо именно поджечь, а не ёбнуться на юпитерное ядро
Аноним 11/01/23 Срд 12:20:22 739765 204
>>739764
>но они ведь не взрываются с огнём
Погляди на видео челябинского метеорита. Да они не взрываются в смысле химической реакции горения, но при прохождении атмосферы выделяется огромное количество тепла.
Аноним 11/01/23 Срд 12:45:43 739766 205
image.png 1200Кб, 1600x900
1600x900
>>739764
Ядерные бомбы тоже без огня взрываются (если пренебречь сколькими-то килограммами химической взрывчатки) если что.
Вопрос в концентрации и количестве энергии, а у крупных метеоритов энергии выделяется очень немало.
В случае со взрывом на Юпитере - огня и так не будет, т.к. окислителя нет (кроме того что с собой привезешь).
А в случае с падением метеорита на землю - и огонь будет, т.к. высокие энергии = высокие температуры, и это поджигает все что горит в окрестностях.
Аноним 11/01/23 Срд 13:20:19 739767 206
>>739751
Даже если тысячу бомб разместить прям в ядре и взорвать одновременно, реакция пойдёт но быстро (за секунды) угаснет. Потому что мало давления и плотность слишком низкая в ядре. Низкая плотность -> медленно идёт реакция -> чтобы шла быстрее нужно разогреть -> разогреваем -> плотность падает -> реакция затухает
Аноним 11/01/23 Срд 14:25:20 739769 207
>>739767
оно поэтому и ядро, что пиздец плотное, металический же водород вроде.
>>739766
ну пиздец, и чё делать? Может к бонбе пару цистерн с жидким кислородом присобачить?
Аноним 11/01/23 Срд 14:46:33 739773 208
>>739769
>ну пиздец, и чё делать?
Чтобы поджечь Юпитерь? Маленькую черную дыру на него сбрось. Аккреционный диск разогреется до температур синтеза. Наверное.
>Может к бонбе пару цистерн с жидким кислородом присобачить?
Чтоб весь юпитер сгорел тебе надо столько же кислорода, сколько весит Юпитер. В итоге у тебя будет газовый гигант с кучей воды и со2 массой в два Юпитера. Гореть, кстати, в процессе тускло будет, вряд ли особо ярче чем сейчас на небе будет выглядеть.
Аноним 11/01/23 Срд 14:58:02 739774 209
а если сжать юпитер до футбольного мяча, бахнет?
Аноним 11/01/23 Срд 15:35:31 739776 210
Аноним 11/01/23 Срд 15:36:26 739777 211
>>739774
>>739776
А, нет, пизжу. Радиус Шварценбургера для Юпитера это где-то три метра. Не бахнет.
Аноним 11/01/23 Срд 17:18:03 739782 212
>>739773
вобщем в пизду юпитер, будем поджигать сатурн, у него диск есть
Какая максимально возможная средняя мощность геомагнитного электрического генератора? fat_alien 11/01/23 Срд 17:36:29 739784 213
image.png 525Кб, 847x1275
847x1275
Индукция магнитного поля Земли меняется от 25 до 65 мкт на поверхности. Следовательно, магнитная энергия примерно 1017,5...18,5 Дж. Общие изменения магнитного поля Земли составляют проценты в день.
Таким образом, средняя мощность составляет 3...300 ГВт. Это нужна только катушка вокруг экватора для производства электроэнергии.

Какая максимально возможная средняя мощность геомагнитного электрического генератора?

Или помогите получить ответ в англоязычной среде https://physics.stackexchange.com/q/721445/327171
Аноним 11/01/23 Срд 18:11:35 739786 214
>>739784
Нахуй нам твой генаратор не впёрнулся, мы тут планеты взрываем, блядь.
Аноним 11/01/23 Срд 18:37:14 739790 215
image.png 218Кб, 480x360
480x360
>>739786
Взрывалка не доросла планеты взрывать. Сперва развиться до галактической империи надо, потом говорить.
Аноним 11/01/23 Срд 20:49:50 739813 216
>>739790
дурное дело не хитрое, всё у нас получится!
Аноним 11/01/23 Срд 22:28:51 739823 217
если Солнце сжать до размеров Луны, его гравитация изменится? И повлияет ли это на орбиты планет? Или похер, и главное масса а не размер тела в пространстве?
Аноним 11/01/23 Срд 22:40:30 739828 218
>>739823
>его гравитация изменится
нет
>И повлияет ли это на орбиты планет
нет
>Или похер, и главное масса а не размер тела в пространстве?
для далеких объектов - похуй
Аноним 12/01/23 Чтв 01:20:07 739847 219
ГЛУШКО ЗЛОДЕЙ МУДАК
КУЗНЕЦОВ САПОГ ВОЯК
КОРОЛАВ КУЛ ЧЕЛ ТАК
Аноним 12/01/23 Чтв 01:21:20 739848 220
>>739847
Фон БраУн - фашист и зигун
Аноним 12/01/23 Чтв 01:59:40 739849 221
>>739823
>Или похер, и главное масса а не размер тела в пространстве?
Да, именно так. Если Солнце заменить на черную дыру массы Солнца, то орбиты планет никак не поменяются.
Аноним 12/01/23 Чтв 02:44:38 739850 222
а вы знали что на юпитере шовной воняет
Аноним 12/01/23 Чтв 06:48:56 739853 223
Для наблюдателя падение объекта в чд сначала ускоряется, а возле гс замедляется. Так вот, какова наибольшая фиксируемая наблюдателем скорость объекта и от чего она зависит: массы чд, положения наблюдателя, может, размера объекта?
Аноним 12/01/23 Чтв 12:26:01 739899 224
>>739784
актуальный вопрос
Аноним 12/01/23 Чтв 15:33:46 739914 225
Привет анон, а подскажи с чего начать свое погружение в астраномию, такую любительскую, интересную с выездом для наблюдений за звездами в малозасвеченные области страны
Аноним 12/01/23 Чтв 20:10:17 739951 226
Когда наша галактика столкнется с Андромедой,то черные дыры в центрах могут завестись на полную катушку и стать квазарами?
Аноним 12/01/23 Чтв 20:42:01 739952 227
>>739951
Не, квазары были на ранних этапах. Просто поактивнее будут чем сейчас, говна на диск насыпется, но не будут(-ет) от пуза отжираться как можно было раньше.
Аноним 12/01/23 Чтв 22:42:16 739971 228
кто прописал и придумал законы в нашей вселенной?
Аноним 12/01/23 Чтв 22:43:53 739974 229
>>739971
Рандомно появились.
Аноним 12/01/23 Чтв 22:52:08 739977 230
SVID20230112214[...].mp4 17100Кб, 1280x720, 00:01:07
1280x720
Аноним 12/01/23 Чтв 23:02:12 739980 231
Аноны, тут мы получили фото туманности NGC 346 с телескопа ДВ. А много там газа осталось для звездообразвания? Лично на мой взгляд, она как то уже исчерпала себя или так выглядит большинство туманностей? Вообще во вселенной сейчас образуются новые туманности?
Аноним 12/01/23 Чтв 23:04:37 739981 232
Аноним 12/01/23 Чтв 23:16:43 739985 233
>>739971
>кто прописал и придумал законы в нашей вселенной?
Кто только не прописывал, кто только не придумывал. На Земле только десятки таких, начиная с хаммурапей заканчивая ньютонами.
Аноним 12/01/23 Чтв 23:27:40 739988 234
>>739981
Почему? Возможна только та вселенная, которая допускает существование наблюдателя. Может в предыдущих не было ничего.
Аноним 13/01/23 Птн 00:26:39 739990 235
Аноним 13/01/23 Птн 00:43:46 739991 236
>>739981
> не бывает так
почему тогда у тебя ещё хуй на лбу не вырос?
Аноним 13/01/23 Птн 00:46:15 739992 237
>>739988
дебс ты
повторяешь чето
ты дебс
Аноним 13/01/23 Птн 00:53:20 739993 238
Каким были Малое и Большое Магеллановые Облака до того как начали поглощаться нашей галактикой?
Местная группа галактик была всегда или все ее члены стали таковыми случайно в результате приливных действий.
Когда Андромеда и Млечный путь сольются и допустим появится еще одна галактика подобной Андромеде, они сольется с нашей или опять обе разрушатся и придется обьединяться заново?
Столкновение галактик это плохо? Вот у нас потихоньку идет звездообразование, а тут как все вспыхнет. В итоге когда в одной не столкнувшейся галактике будет 100 новых и 150 старых звезд, у нас будет 200 старых и 50 новых? Я правильно понимаю что это как поджечь склад пиротехники. Красиво, но быстро.
Аноним 13/01/23 Птн 00:57:02 739994 239
>>739971
никто не знает и практически наверняка никогда не узнает
Аноним 13/01/23 Птн 01:17:20 739995 240
>>739993
> Каким были Малое и Большое Магеллановые Облака до того как начали поглощаться нашей галактикой?
были несколько более правильной формы, не спиральными или дисковыми
> Местная группа галактик была всегда или все ее члены стали таковыми случайно в результате
ну относительные скорости не очень высокие, скорее всего изначально были недалеко
> приливных действий.
приливные действия это про другое, просто тяготения
> Когда Андромеда и Млечный путь сольются и допустим появится еще одна галактика подобной Андромеде, они сольется с нашей или опять обе разрушатся и придется обьединяться заново?
слишком косноязычно, кто они то блядь
> Столкновение галактик это плохо?
плохо относительно кого чего, для какой задачи?
Аноним 13/01/23 Птн 01:28:06 739996 241
>>739784
кто-то где-то здесь считал сколько нужно гигаватт и медяхи для производства магнитосферы на марсе, но было это давненько
Аноним 13/01/23 Птн 01:29:59 739997 242
>>739996
Помню тебя, был в том треде.

мимо алкаш
Аноним 13/01/23 Птн 01:40:00 739998 243
>>739503
> то особо-то проблем для создания искусственной планеты вокруг чёрной дыры с быть не должно?
нет, это классический ответ на сферы дайсона, прочности никаких материалов не хватит, разве что дырка будет весом в не самый крупный астероид, только кольца или системы колец
Аноним 13/01/23 Птн 02:05:44 739999 244
>>739995
>слишком косноязычно, кто они то блядь
Ну не они, она. Опечатался. Наша будущая галактика и еще одна равная по массе и размеру Андромеде. Она сможет разорвать такую Андромеду x и поглотить ее или все будет по старому сценарию?
>плохо относительно кого чего, для какой задачи?
Во времени. Галактика образованная таким образом быстрее стареет?
Аноним 13/01/23 Птн 05:05:55 740001 245
>>739971
Никто, нету никаких "законов Вселенной". Законы придумали люди, чтобы описывать наблюдаемую реальность. А реальность такая, какая есть, существующая сама по себе, независимо от сознания. Мы можем эту реальность лишь изучать, выявлять в ней определенные закономерности и на их основе придумывать "законы физики", с помощью которых мы можем адекватно описывать окружающую реальность. Самих по себе "законов" не существует, они нигде и никем не прописаны.
Аноним 13/01/23 Птн 05:18:31 740002 246
131231231231231[...].jpg 189Кб, 1700x2164
1700x2164
Мы еще получаем информацию с Вояджеров, а почему нельзя еще такой же запустить в облако Оорта. Технологии сильно продвинулись вперед. И наверное будет ответ, что у нас уже есть Вояджеры и я отвечу. Он сука только лет через 30 начнет покидать границы СС, зачем откладывать этот момент?
И тут я посмотрел на туманность Конская голова, я не вижу конскую голову. Я вижу огромную человекоподобную фигуру с 2 руками, правда без головы и она возвышается над всей остальной туманностью. Может кто ее обрисовать может, я не вижу конскую голову.
Аноним 13/01/23 Птн 06:35:58 740005 247
>>739998
А если регулярно говны на чд скидывать, не будет ли тебя довлением поддерживать на лету?
Аноним 13/01/23 Птн 07:34:40 740007 248
>>739993
>Когда Андромеда и Млечный путь сольются и допустим появится еще одна галактика подобной Андромеде
Когда они сольются, образуется большая эллиптическая галактика. Потом с этой галактикой сольются все карликовые галактики Местной группы. И будет одна гигантская эллиптическая галактика.
Аноним 13/01/23 Птн 09:27:53 740011 249
>>739988
>Возможна только та вселенная, которая допускает существование наблюдателя.
Нет никаких "наблюдателей" как отдельных особых сущностей, от которых зависит существование аж целой Вселенной. Вселенной насрать, наблюдает её кто-то или нет. Как существовала 13 миллиардов лет без нас если мы первые и единственные, так и продолжить существовать триллионы триллионов лет и без "наблюдателей".
Аноним 13/01/23 Птн 10:15:53 740019 250
>>740007
>гигантская
чел "все карликовые галактики" в сумме и половины звёзд не прибавят, какая тут "гигантская" лол
Аноним 13/01/23 Птн 10:22:45 740020 251
>>740019
Эллиптические галактики гигантские сами по себе.
Аноним 13/01/23 Птн 10:27:58 740021 252
image.png 496Кб, 616x353
616x353
А возможна ли простейшая самоподдерживающаяся автоматизация?
Скажем, роботы+автоматы чтобы обрабатывали породу, выплавляли металлы, обрабатывали и собирали сами себя?
Если да, то по идее с этого и надо начинать базы на луне/марсе, заслать стартовую коллекцию, они бы большую площадь перелопатили и собрали бы модулей/запчастей для людей-посетителей уже.
Аноним 13/01/23 Птн 10:56:03 740027 253
>>740021
Ты сейчас зонды фон Неймана. Теоретически, всё что могут людишки, могут и роботы.
Аноним 13/01/23 Птн 11:02:33 740028 254
>>740001
тогда как работает гравитация, притяжение в атомных ядрах и прочие взаимодействия? Просто само по себе - так случайно получилось - по моему херня. Есть даже такое понятие как "Тонкая настройка вселенной". Не верю что все это бахнуло из одной точки и развернулось до вселенной со звездами, планетами и органической жизнью - само по себе и случайно. Без участия каких то инженеров и программистов из вселенной высшего порядка и тд.
Аноним 13/01/23 Птн 11:07:48 740030 255
>>740028
>тогда как работает гравитация, притяжение в атомных ядрах и прочие взаимодействия?
Могу объяснить по теории плотноприжатых фотонов
>Без участия каких то инженеров и программистов из вселенной высшего порядка и тд.
Всё верно - наш Мир виртуальный
БАЗА! БАЗА!! БАЗА!!! Аноним 13/01/23 Птн 11:41:15 740037 256
В основе мироздания лежит xyй размером с Вселенную.

Xyй первичен, а пиздa вторична.
В основе всего лежит бинарный код, где крепкий xyeц это кол, единица, сущее, а пиздa это дырка, нуль, войд, ниxyя, пропасть, отсутсвие бытия. 1 и 0.

Тот факт, что самки это ходячие инкубаторы по высиранию из матки новых пиплов, не отменяет того, что они ноль без палочки.

Существует только один xyй размером с Вселенную. Можешь называть его Богом, если тебе так удобно.
Аноним 13/01/23 Птн 12:16:19 740043 257
131231231231231[...].jpg 431Кб, 1700x2164
1700x2164
>>740002
Когда ее открыли телескопы похуже были.
Аноним 13/01/23 Птн 12:20:05 740044 258
>>740021
>Скажем, роботы+автоматы чтобы обрабатывали породу, выплавляли металлы, обрабатывали и собирали сами себя?
Про это даже книжку написали, "Крабы идут по острову" называется. Как и бывает в книжках на такою тему кончилось там все плохо.
Аноним 13/01/23 Птн 12:21:20 740045 259
>>740030
>Могу объяснить по теории плотноприжатых фотонов
Я уж испугался, что тебя отпустило и ты нас больше не порадуешь.
Аноним 13/01/23 Птн 12:23:45 740046 260
>>740028
Ошибка выжившего.
Аноним 13/01/23 Птн 12:41:08 740049 261
>>740021
Вполне возможна. Ведь органическая жизнь это упорядоченное нагромождение наномашин.
>роботы+автоматы чтобы обрабатывали породу, выплавляли металлы, обрабатывали и собирали сами себя?
Сейчас идет тренд на полное вытеснение человека из производительных цепочек и полной автоматизации производства, даже систему диагностики и ремонта.
>по идее с этого и надо начинать базы на луне/марсе
Об этом думали еще до твоего рождения, но пока что цена мясного мешка меньше супер продвинутого робота. А на инопланетной чужбине еще надо целый завод собирать, что и на родине не всегда получается.
Аноним 13/01/23 Птн 13:23:00 740055 262
>>740028
>тогда как работает гравитация, притяжение в атомных ядрах и прочие взаимодействия?
Если ты говоришь о фундаментальных константах нашей Вселенной, то в них ничего особенного нет. Современная космология оперирует понятием мультиверса. Мультиверс включает колоссальное множество вселенных с разными значениями констант. Среди такого множества вселенных неизбежно найдется вселенная с подходящими для существования разумной жизни константами. Мы как раз живем в одной из таких. Вот тебе и вся "тонкая настройка".
>Не верю что все это бахнуло из одной точки
Не было никакой "точки" и нихуя нигде не бахнуло. Откуда вы берете эту хуйню? Большой взрыв совсем не так работает.
>Без участия каких то инженеров и программистов
Лишние сущности, которые нахуй не нужны. Ландшафт теории струн и инфляционный мультиверс позволяют сколько угодно спаунить вселенные с разными наборами констант чисто рандомным путем.
Идея разумного дизайна в любом виде ущербна в своей основе, поскольку неизбежно порождает бесконечные цепочки создателей и творений. Т.е. она вообще нихуя не объясняет, городя бесконечное число бесполезных сущностей. Оставим эту идею церковным бабкам.
Аноним 13/01/23 Птн 14:10:39 740058 263
image.png 984Кб, 1024x581
1024x581
>>740027
>>740044
>>740049
А кто-то проводил научно-исследовательские изыскания по данному вопросу? Скажем, количество операций необходимое для создания производственной базы способной провести такое количество операций, какие это операции и как выполняются, разложить на составные, получить что надо печь, тигли, манипуляторы в количестве н штук, прессы в количестве к штук, полуавтономные дроны по сбору реголита, солнечные панели и так далее, и сколько бы это весило с расчетом того что бы можно было это закинуть на марс и сказать "похеали"?
Может посчитали и на текущем уровне это реально, может даже прототип втихую запилили, но весить будет пять тысяч тонн и мы просто столько не потянем на марс закидывать?
Я просто фантазирую, что такую фабрику закинуть куда-нибудь в Элладу, она себя задуплит раз пятьдесят, а потом уже с распечатанными запчастями будет собирать роверы геологоразведки ареологоразведки, системы жизнеобеспечения, бараки и так далее, м-м-м.
Инб4 марс не нужен, нахуй гравитационные ямы, надо жить в космосе
Аноним 13/01/23 Птн 14:21:06 740059 264
>>740058
>какие это операции и как выполняются, разложить на составные
Нужно буквально создать всю индустриальную цепочку, вот буквально всю что существует в современном индустриальном обществе.
>закинуть куда-нибудь в Элладу
Думаю даже если ее закинуть на земную Элладу, она долго не проработает без вмешательства из вне.
Аноним 13/01/23 Птн 14:51:06 740062 265
>>740055
> Ландшафт теории струн и инфляционный мультиверс
обе эти хуйни были были придуманы как предполагаемая функциональная зависимость позволяющая получить более точные параметры текущей вселенной из антропного принципа (но в реальности никаких работ по выяснению этих параметров нет)
> позволяют сколько угодно спаунить вселенные с разными наборами констант чисто рандомным путем.
> Т.е. она вообще нихуя не объясняет, городя бесконечное число бесполезных сущностей.
иронично
Аноним 13/01/23 Птн 15:07:31 740063 266
>>740058
Проводили и сейчас проводятся, даже есть целая парадигма.
Индустрия в космосе очень даже возможна, однако есть много подводных камней.
В первую очередь для этого надо развивать соответствующие технологии дальше чем в лаборатории на коленке энтузиаста, а тут большие проблемы, ибо энтузиастов мало и они часто перетекают в коммерцию.
Второе это политика. Людям проще друг друга убивать, чем обстроить солнечную электростанцию в Сахаре, а ты хочешь чтоб на сранной луне в реголите копались.
Третье, нет альтернативе человеку как универсальной рабочей единице. ИИ даже на земле еще не прижился.
Аноним 13/01/23 Птн 15:11:08 740064 267
Аноним 13/01/23 Птн 15:19:06 740066 268
>>740055
так мультиверс это такая же фантазия как и бог на небесах. Никаких доказательств на эту тему нет. А теория струн это вообще математика сплошная, и не факт что в реальности эти струны и браны существуют. Если нет ничего уникального в том что из водорода и гелия в конечном итоге возникли люди, то я вообще тогда не знаю что считать уникальным. Вектор развития в любом случае был чем то задан, или кем то. И возникновение жизни тоже было изначально заложено, иначе бы ее не возникло. А списывать все на набор случайностей по моему глупо. Предпосылки для этих случайностей были с самого начала.
Аноним 13/01/23 Птн 15:25:06 740067 269
Как из водорода и гелия образовались другие хим элементы?
Аноним 13/01/23 Птн 15:28:10 740068 270
>>740067
Термоядерный синтез, едрить его за ногу!
Аноним 13/01/23 Птн 15:43:13 740069 271
>>740067
Три ядра гелия сливаются в углерод, а далее углерод последовательно захватывает протоны пока не дойдет до кислорода и распадается до углерода и гелия.
При жестких условиях гелий и кислород могут слиться в неон, а он уже с другим ядром гелия в магний, и так до железа. Это называют альфа процессом. В нем иногда выделяются нейтроны или гамма кванты. Первые захватываются и получатся промежуточные элементы, а так еж ядра тяжелее железа, а гаммы кванты активируют другие ядра, производя нейтроны.
Сверхтяжелые ядра уже производятся во всяких взрывных событиях, вроде слияние белых карликов/нейтронных звезд при которых часть вещества выбрасывается, или при аккреции вещества на релятивистские объекты. Всякие джеты довольно богатые тяжелыми элементами.
Аноним 13/01/23 Птн 16:05:34 740070 272
Какова вероятность,что ЧД из за своих свойств содержат неизвестный хим элемент?
Аноним 13/01/23 Птн 16:13:15 740071 273
>>740070
> хим элемент?
> хим
нулевая, химия внутри чд не работает
Аноним 13/01/23 Птн 16:14:45 740072 274
>>740071
Мы ничего не знаем о том,что там внутри.
Есть теория что это гравастар
Аноним 13/01/23 Птн 16:29:02 740073 275
>>740072
ЭМ взаимодействия не работают в чд, собственно это прямо следует из определения чд, фотон не может перейти на более высокую орбиту внутри чд, гравастар это попытка отрицать сингулярность планковскими величинами, которым приписали сакральный смысл, впрочем это никак не отрицает основополагающего свойства чд
Аноним 13/01/23 Птн 17:09:12 740077 276
>>740058
>А кто-то проводил научно-исследовательские изыскания по данному вопросу?
Ну подсчитывали, что даже без сверхсвета такие машины должны распространиться по всей нашей галактике всего за полмиллиона лет. Т.е. опять упёрлись в парадокс Ферми, потому что наша галактика существует намного дольше, но в нашей системе зонды что-то не наблюдаются, и следов их деятельности не видно. Т.е. либо с зондами всё не так просто либо мы и есть своего рода зонды фон Неймана.
Аноним 13/01/23 Птн 17:15:07 740078 277
>>740002
>Я вижу огромную человекоподобную фигуру с 2 руками, правда без головы и она возвышается над всей остальной туманностью.
Нет, там обезглавленная русалка вынырнула из бушующего моря.
Аноним 13/01/23 Птн 19:42:16 740091 278
image.png 81Кб, 198x255
198x255
>>740002
Есть разрешение еще выше четкостью, там вообще лошадью не пахнет, как ни щурься. А вот на старых фотках было видно лошадку как на шахматной фигуре.
Аноним 13/01/23 Птн 21:54:52 740107 279
Валеру обманули[...].mp4 1554Кб, 198x360, 00:00:45
198x360
Почему в стакане только половина, если его перевернуть???
Аноним 14/01/23 Суб 00:05:53 740115 280
>>739847
Я сын Глушко!
РД - круто!
Аноним 14/01/23 Суб 00:06:38 740116 281
16736303315620.jpg 1085Кб, 1218x2048
1218x2048
Аноним 14/01/23 Суб 00:32:49 740117 282
Метановые ракеты когда?
Аноним 14/01/23 Суб 02:44:50 740128 283
>>739914
> Привет анон, а подскажи с чего начать свое погружение в астраномию,
С теории.
> такую любительскую, интересную с выездом для наблюдений за звездами в малозасвеченные области страны
Телескопов тред.
>>740107
Стакан не представляет собой цилиндр, это перевёрнутый усеченный конус.
>>740117
Завтра.
Аноним 14/01/23 Суб 06:29:34 740130 284
>>740062
>обе эти хуйни были были придуманы как предполагаемая функциональная зависимость
Ты нагородил набор слов.
Ландшафт теории струн и инфляционный мультиверс просто дают естественный ответ на вопрос о "тонкой настройке". И если еще до теории струн можно доебаться, что она имеет неизвестное отношение к реальности, т.к. не дает никаких проверяемых предсказаний, ее статус сейчас вообще не ясен, то до теории инфляции доебки выглядят максимально уныло и глупо. Теория инфляции хорошо подтверждается целой массой наблюдательных данных, без нее современная космология уже не может обойтись - она встроена во многие вещи. Ну а теория струн сама по себе является стройной математической схемой, которая дает красивые непротиворечивые объяснения множеству физических фактов. В частности, дает красивое объяснение т.н. тонкой настройке.
>иронично
Хуично. Мультиверс легко и непринужденно позволяет объяснить "тонкую настройку", с которой веруны носятся как с писаной торбой. Неограниченно рождающиеся вселенные в ходе инфляции естественно и логично вытекают из современной инфляционной космологии, которая базируется на точных данных космического микроволнового фона. Т.е. мультиверс - неотъемлемая часть современной науки о Вселенной. А поповский пиздеж про "программистов вселенной" - это просто поповский пиздеж.
>>740066
>так мультиверс это такая же фантазия как и бог на небесах
Мультиверс - неотъемлемая часть современной космологии. Без него нихуя не сходится и не работает. А современная космология подтверждается точнейшими данными космического микроволнового фона. Оставь свою дешевую демагогию тем церковным бабкам, у которых ты нахватался чепухи про "программиста вселенной".
>А теория струн это вообще математика сплошная
Математика - язык науки. Без математики науки не существует. Теория струн - непротиворечивая математическая схема, которая может объяснить многие вещи. Твой пиздеж про "программиста" не опирается ни на какую математику и нихуя не может объяснить. Как думаешь, что выглядит предпочтительнее?
>и не факт что в реальности эти струны и браны существуют
Вообще не факт. Но для генерации вселенных с отличающимися фундаментальными константами существует и иной механизм, для которого никакая теория струн не нужна. Я привел струнный ландшафт лишь как наиболее популярный и известный пример.
>Если нет ничего уникального в том что из водорода и гелия в конечном итоге возникли люди
Ну конечно же так захотел какой-то бох, твердо и четко.
Уникального в этом нет нихуя, поскольку среди огромного множества вселенных чисто статистически неизбежно будет найдена вселенная с таким набором констант, который подойдет для возникновения разумной жизни. Больше нихуя не надо. Никакой бох не нужен. Оставь его для песнопений в церкви.
>Вектор развития в любом случае был чем то задан, или кем то
Ты скозал?
>И возникновение жизни тоже было изначально заложено, иначе бы ее не возникло
Ебать у тебя логика. Возникновение жизни - это просто одно из триллионов событий во Вселенной. Могло произойти, а могло не произойти. Условия для этого во Вселенной сложились - оно произошло. Во вселенных, где таких условий нет, просто-напросто никто такими вопросами не задается - их некому задавать. Вот тебе и весь "вектор развития".
>А списывать все на набор случайностей по моему глупо
Верунок ебаный, у нас вся жизнь - это набор случайностей. С добрым утром. Хотя когда у тебя мышление церковной бабки, то весь мир становится преисполнен чудесами, тут без сомнений.
>Предпосылки для этих случайностей были с самого начала
Если речь про зарождение жизни и появление человека, то предпосылка тут единственная - статистически обусловленное появление в мультиверсе вселенной с подходящим набором фундаментальных констант. В такой Вселенной мы и живем. Ничего удивительного и сверхъестественного здесь нет.
>>740070
В ЧД нет никаких химических элементов и быть там не может по определению.
Аноним 14/01/23 Суб 06:52:44 740132 285
>>740128
>Стакан не представляет собой цилиндр, это перевёрнутый усеченный конус.
Стакан топологически является сферой. А вот дырявая ложка это тор.
Аноним 14/01/23 Суб 12:27:32 740143 286
>>740130
>никаких химических элементов
Ты сказал?
Аноним 14/01/23 Суб 14:32:33 740148 287
Аноним 14/01/23 Суб 14:42:55 740150 288
>>740148
НИКТО не знает,что там происходит
Аноним 14/01/23 Суб 16:04:04 740160 289
>>740150
Никто не знает, что происходит в сингулярности. Кроме того, что вещество, попадая туда, разрушается, как и само пространство-время, преобразуясь в какую-то иную форму материи. В какую - хуй знает. Но из известного точно можно сказать, что никаких химических элементов там не может существовать по определению, т.к. еще при подлете к сингулярности вещество разрывается до состояния элементарных частиц.
Аноним 14/01/23 Суб 16:41:59 740162 290
>>740130
> дают естественный ответ на вопрос о "тонкой настройке".
> Мультиверс легко и непринужденно позволяет объяснить "тонкую настройку"
нефальсифицируемыми вскукареками ничего объяснить нельзя
> мультиверс - неотъемлемая часть современной науки о Вселенной
мультиверс вообще никакого отношения к науке не имеет
> Неограниченно рождающиеся вселенные в ходе инфляции естественно и логично вытекают из современной инфляционной космологии, которая базируется на точных данных космического микроволнового фона.
ты нагородил набор слов, из микроволнового фона никаких мультивселенных не следует
Аноним 14/01/23 Суб 22:34:55 740194 291
image.png 1630Кб, 1332x850
1332x850
image.png 3450Кб, 1512x2162
1512x2162
image.png 2827Кб, 1920x1200
1920x1200
image.png 2620Кб, 1920x1080
1920x1080
Появился вопрос по планетам-пустыням.
Вот, например, арты. Что мы на них видим? Просторы, камни, ветровая эрозия, скалы, пески.
Так?
А откуда может взяться песок, если так подумать? Откуда он берётся, для образования пустынь? Песок же, типа, результат действия морей и океанов на сушу бывшего дна под толщей воды. А если планета - пустыня, то откуда там вода и куда она пропадёт?
Если нет вод, то и скал, и песков быть не может, только пологие горы. Никаких песчаных дюн, никакого Арракиса или Татуина, получается?
Или вот пик2: красиво, интересно, но как такое может быть?
Как вообще тогда может быть климат стабильным на планете, где только гулй-ветер и нету влаги, образующие облака и вообще погоду
Аноним 15/01/23 Вск 00:06:22 740209 292
>>740194
Ну ты подумой хоть немножко, ответ же прямо перед глазами
Аноним 15/01/23 Вск 05:18:54 740211 293
1673749134727.png 473Кб, 1420x1626
1420x1626
>>740072
> Есть теория что это гравастар
Аноним 15/01/23 Вск 05:26:08 740212 294
>>740194
> А откуда может взяться песок, если так подумать?
из пизды любой девушки которая увидит твой шизоидный бред и у нее начнётся эрозия
Аноним 15/01/23 Вск 06:44:29 740213 295
>>740194
>Песок же, типа, результат действия морей и океанов
Песок возникает от разрушения горных пород неважно чем - водой, перепадами температур или ветром.
то и скал
Скалам вода тоже нге нужна, хватит и тектонических процессов.
>Как вообще тогда может быть климат стабильным на планете
Никак, будет как в земных пыстынях или на Марсе - днем жарко, ночью холодно, летом жарища, зимой дубак.

Да и кислороду на планете пустыне неоткуда взяться.
Аноним 15/01/23 Вск 20:18:29 740270 296
Какая была бы температура на Венере, если бы у неё была бы атмосфера такая же, как у Земли? А на Марсе?
Аноним 15/01/23 Вск 21:19:00 740272 297
>>740270
На Марсе можно было бы вести хозяйственную деятельность во впадинах на экваторе и в Хелласе. Даже купатся можно было бы летом в мелких озёрах. На Венере высокие широты (скажем, 70+) были бы пригодны для жизни, но только если климат сухой, пустынный
Аноним 15/01/23 Вск 21:40:45 740273 298
>>740270
На Марсе делать нечего
Аноним 15/01/23 Вск 21:48:53 740276 299
Слетать и высадиться на Меркурий это совсем за гранью фантастики? Или в будущем есть шансы? В каких нить жаропрочных костюмах на теневой стороне планеты например.
Аноним 15/01/23 Вск 21:51:06 740277 300
Как появились газовые планеты гиганты?
Аноним 15/01/23 Вск 21:53:18 740279 301
>>740277
как обычно
>>740276
на обратной стороне не жарко, основная проблема в двигателях, нужно очень много дельты
Аноним 15/01/23 Вск 21:55:31 740280 302
Когда Солнце раздуется,что будет с ледяными гигантами?
Аноним 15/01/23 Вск 22:08:12 740281 303
Аноним 15/01/23 Вск 22:10:08 740283 304
>>740281
Ну лед и жидкие газы испаряться,и че останется голое ядро-камень?
Аноним 15/01/23 Вск 22:47:10 740284 305
>>740283
Нет.
Ледяные гиганты все равно слишком далеко будут от солнца.
Они всего лишь нагреются и чуть распухнут.
Аноним 16/01/23 Пнд 06:35:28 740313 306
>>740277
На голое твердое ядро налепилась толстая газовая оболочка, которая не сдувается солнечным ветром и не испаряется, потому что находится на достаточном удалении от звезды, за снеговой линией.
Аноним 16/01/23 Пнд 14:10:10 740340 307
Аноним 16/01/23 Пнд 15:27:30 740354 308
>>740340
Что там? Не хочу ходить по сомнительным ссылкам
Аноним 16/01/23 Пнд 15:34:59 740355 309
>>740354
Какие-то карикатурные уфологи с тарелочками.
Аноним 16/01/23 Пнд 22:29:43 740383 310
167389714101133[...].jpg 269Кб, 725x1000
725x1000
167389736495894[...].jpg 243Кб, 857x1100
857x1100
167389641416261[...].jpg 229Кб, 1000x848
1000x848
167389646954380[...].jpg 232Кб, 843x1100
843x1100
Неуверен постили уже тут или нет, прошу знающих дать свое мнение о пиках, имеет ли это хоть какое либо адекватное обоснование или все это шиза уровня /zog
Аноним 16/01/23 Пнд 22:37:58 740385 311
>>740383
Сложно, надо думать. Мне сейчас лень (устал), а кроме меня тут умных нет, только "популярные" дебичи которые сейчас тебе ответят популярно, с наукообразными научпоп-словами...ну ты понял
Аноним 16/01/23 Пнд 22:44:05 740386 312
>>740383
> шиза уровня /zog
Это.
Хуй, который тебе ответил надо мной, кстати, тоже.
Аноним 16/01/23 Пнд 22:51:53 740387 313
>>740383
Псевдонаучная дичь.
Аноним 16/01/23 Пнд 23:14:37 740388 314
>>740383
>какие-то фантазии без капли обоснования
Сам-то как думаешь?
с плотноприжатых сфер обосрался
Аноним 16/01/23 Пнд 23:26:50 740389 315
Преоны считаются опровергнутыми уже, или надо коллайдер побольше?
Аноним 17/01/23 Втр 00:25:48 740392 316
>>740389
Они -- причина коровьего бешенства.
Аноним 17/01/23 Втр 01:49:37 740396 317
image.png 345Кб, 600x412
600x412
Аноним 17/01/23 Втр 02:01:42 740398 318
>>740397
До 1% СО2 в атмосфере не грозит долгосрочными проблемами (если не считать падение производительности). Там общее давление 1 бар было же?
>В присланных подарках нашёл фильм ... Посмотрел
На чем фильм был? Видеокассэту буржуинскую что ли притаранили? Или кинопроектор приебенили с бобиною?
Аноним 17/01/23 Втр 03:18:49 740405 319
>>740389
какого-то обоснования кроме "красивости" в противовес хиггсовскому механизму не было, так что опровергать нечего
в общем смысле что нужно копать структуру глубже - вполне себе живо в петлях и струнах, но коллайдоры да, просят размером с солнечную систему
Аноним 17/01/23 Втр 04:39:07 740410 320
Какой-то ебнутый итт мочератор.
Аноним 17/01/23 Втр 08:47:24 740414 321
image.png 226Кб, 1000x1000
1000x1000
>>740355
>Какие-то карикатурные уфологи с тарелочками.
Аноним 17/01/23 Втр 08:53:22 740415 322
>>740383
Инфляционный мультиверс - неотъемлемая часть современной космологии, так что это более чем обосновано. Именно в космологической части. Многомировая же интерпретация квантовой механики - чистые спекуляции, пиздеж ни о чем.
>>740389
Это классическая хуйня без задач. Никто никаких "преонов" нигде не видел и никогда не увидит. В теоретической физике достаточно подобных мусорных идей.
Аноним 17/01/23 Втр 08:57:12 740416 323
perseverans-3.jpg 171Кб, 1402x791
1402x791
Нахуя нужна это сложная работа с доставкой пробы грунта с марсохода Perseverance , к тому же бур там всего на 10 см за сотни миллионов лет не занесло все марсианскими бурями? Чего они хотят найти там на такой малой глубине? Там нихуя не будет, за миллиард лет нужно гораздо больше слой снимать, больше инфы даст марсианские метеориты которые попали от метеоритного столкновения с марсом и занесённые на землю.
Аноним 17/01/23 Втр 09:18:21 740417 324
image.png 161Кб, 383x488
383x488
>>740273
там можно срать в песок и сажать в гавно картошку, можно скалы хуями изрисовать, можно бомбу ядрёную скинуть в олимп и зырять чё произойдёт. Да можно дохуя чё делать, если делать нечего. Ивана гамаза туда отправить, он специалист по "советам от скуки"
Аноним 17/01/23 Втр 09:19:36 740418 325
>>740284
потеряют много газиков
Аноним 17/01/23 Втр 09:52:45 740419 326
image.png 30Кб, 120x137
120x137
>>740383
это почему это в паралельной вселенной люди четырёхрукие? это какойто блядский стереотип
Аноним 17/01/23 Втр 12:29:31 740436 327
>>740418
Вряд ли, они скорее даже наоборот могут прибалять в массе, поскольку на этом этапе Солнце будет обильно срать медленными газом и пылью, которые будут успешно захватывается магнитным и гравитационным полем планет.
В добавок Солнце на этапе красного гиганта уже будет значительно слабо излучать в УФ - основного фактора фотоиспарения атмосфер.
Аноним 17/01/23 Втр 12:30:03 740437 328
>>740419
Хорошо, что у них не пять хуйцов.
Аноним 17/01/23 Втр 12:32:41 740438 329
>>740416
Геологи не наугад бурят, а выбирают места где необходимые породы находятся в обнаженном состоянии.
Аноним 17/01/23 Втр 13:18:45 740441 330
Неужели и правда у Роскосмоса дела с коммерческими запусками становятся всё хуже и хуже?
Аноним 17/01/23 Втр 13:29:50 740442 331
>>740441
>Неужели и правда у Роскосмоса дела с коммерческими запусками становятся всё хуже и хуже?
Рогозин, ты? А помниш как мы всей европой нассали тебе в …
Аноним 17/01/23 Втр 13:38:29 740445 332
>>740441
Это очевидно и предсказуемо.
Аноним 17/01/23 Втр 18:48:46 740494 333
>>740437
Пистолетов был бы не против, вот бы он захуярил свою мельницу пятью хуями-то!
Аноним 17/01/23 Втр 23:12:18 740514 334
Допустим мне получилось разогнать тенисный мяч до скорости света в атмосфере Земли. Он же просто взорвётся ядерным взрывом от контакта с воздухом не пролетев пары метров?
Аноним 17/01/23 Втр 23:39:18 740516 335
167398719269568[...].jpeg 50Кб, 433x529
433x529
>>740385
>>740386
>>740387
>>740388
>>740415
>>740419
Один говорит одно, другой говорит другое, понятно все с вами, в /б/ содержательней ответы дают, идите нахуй
Аноним 17/01/23 Втр 23:49:33 740518 336
>>740514
Мячик будет как сгусток релятивистских частиц. Космические лучи проникают до поверхности Земли, так что мячик немного испарится но по-большей части долетит до поверхности и врежется, будет атомный взрыв, огромный кратер, энергия в 100 миллионов Гигаджоулей это 25 Мегатонн тротила
Аноним 18/01/23 Срд 00:26:01 740520 337
>>740516
>в /б/
Да, тебе там самое место.
Аноним 18/01/23 Срд 00:26:54 740521 338
>>740514
>>740518
Если буквально со скоростью света, то взрыв будет бесконечной мощности.
Аноним 18/01/23 Срд 03:23:24 740525 339
>>740521
Забавно, что скорость по космическим меркам не такая и большая, т.е. никакого эффекта вечного сокола как в зв когда звёзды смазываются и быть не может, даже с планетами, а все равно может выделиться такая колоссальная энергия.
Аноним 18/01/23 Срд 04:44:03 740526 340
IskanderednaksI[...].jpeg 2582Кб, 3617x3617
3617x3617
Влияет ли химический состав атмосферы на температуру на планете?
Если мы возьмём и заменим все газы поверхности нашей атмосфера на углекислый газ? А если на аргон? А если на водород и гелий? будут ли какие-нибудь парниковые-антипарниковые эффекты?
Аноним 18/01/23 Срд 08:07:38 740528 341
>>740526
>будут ли какие-нибудь парниковые-антипарниковые эффекты?
Будут.
Аноним 18/01/23 Срд 12:41:37 740556 342
>>740283
>Ну лед и жидкие газы испаряться
Тот "лед" из которого состоят эти гиганты, на самом деле нагрет до тысяч градусов
Аноним 18/01/23 Срд 13:03:08 740558 343
Аноним 18/01/23 Срд 13:24:10 740559 344
image.png 161Кб, 1844x1246
1844x1246
Аноним 18/01/23 Срд 13:29:54 740560 345
>>740559
Ну и где тут лёд? Ты обосрался, как и дебич что придумал "ледяной гигант". Есть только газовые гиганты разных масс
Аноним 18/01/23 Срд 20:28:57 740602 346
>>740560
блять, лёд 10 вверху диаграммы, ара у тебя глаз нету?
Аноним 18/01/23 Срд 21:32:44 740606 347
Аноним 18/01/23 Срд 22:26:33 740610 348
Аноним 19/01/23 Чтв 02:10:54 740619 349
А вообще действительно почему скорость света именно 300к сек, именно при этом значении все остальное типа энергии стремиться к бесконечности как в сингулярности, каждая доля процента ускорения при приближении к 100% уже ебанутые энергии. Есть какое-то мат объяснение что именно 300к, иначе другое не сходится или это просто такая рандомная константа, выпавшая нашей вселенной?
Аноним 19/01/23 Чтв 02:33:50 740620 350
>>740602
А теперь давай скажи мне пидорок юродивый какое давление и какая температура под атмосферой твоего ссаного "ледяного гиганта"
Аноним 19/01/23 Чтв 05:19:11 740632 351
>>740276
> Слетать и высадиться на Меркурий это совсем за гранью фантастики?
Нет никакой фантастики. Проблемы почти те же что с Марсом плюс местоположение.
> Или в будущем есть шансы? В каких нить жаропрочных костюмах на теневой стороне планеты например.
Шансы есть, но их нет. Ну т.е. гипотетически слетать можно было бы, и сесть в полярных зонах, но зачем.
>>740410
Здесь нельзя трогать кнопку репортов. Вообще. Только в самом крайнем случае.
>>740526
> Влияет ли химический состав атмосферы на температуру на планете?
Да. Ты только что история земли.
> Если мы возьмём и заменим все газы поверхности нашей атмосфера на углекислый газ?
Ты только что Венера.
> А если на аргон?
Хз возможно ли это.
>А если на водород и гелий?
Это ты так на суперземли намекаешь?
> будут ли какие-нибудь парниковые-антипарниковые эффекты?
С аргоном хз, с гелиево-водородной атмосферой очевидно что парниковый будет.
Аноним 19/01/23 Чтв 05:21:24 740633 352
>>738638 (OP)
>Q: В космосе горячо или холодно?

Температура среды на расстоянии 1 астрономическая единица от Солнца примерно несколько тысяч K.
Аноним 19/01/23 Чтв 05:24:30 740634 353
Вам ничего не кажется странным в этом сообщении?

https://ria.ru/20221208/ksenon-1837268978.html

В России столкнулись с дефицитом ксенона

Спутникостроители отмечают значительный дефицит и подорожание ксенона, что влияет на программы производства космических аппаратов, сообщил РИА Новости в четверг директор калининградского опытного конструкторского бюро (ОКБ) "Факел" Геннадий Абраменков.
Ксенон используется в спутникостроении в качестве топлива для электроракетеых двигательных установок. Газ подвергается воздействию нейтрально заряженных атомов и высвобождает отрицательно заряженные электроны, а сам ионизируется - превращается в положительные ионы. Они разгоняются под воздействием электростатического поля и, вылетая из двигателя, придают спутнику необходимый импульс.
"Ксенон, в первую очередь, сильно подорожал, что влияет на экономику и отдельных аппаратов, и группировки в целом. И второе, наблюдается значительный дефицит, в принципе, ксенона на рынке, то есть необходимого объема для заправки сейчас нет",- сказал Абраменков, отвечая на вопрос о том, какие трудности сейчас испытывают предприятия-производители спутников и как это сказывается на сроках производства.
Аноним 19/01/23 Чтв 06:29:07 740636 354
>>740619
Да, такая константа.
> Есть какое-то мат объяснение что именно 300к, иначе другое не сходится
Это то что мы наблюдаем.
Аноним 19/01/23 Чтв 07:05:48 740638 355
>>740634
А что странного? Что у самой большой страны в мире ксенона должно быть хоть жопой жуй? Или что?
Аноним 19/01/23 Чтв 07:10:54 740639 356
>>740638
>А что странного?

"Ксенон используется в спутникостроении в качестве топлива для электроракетеых двигательных установок. Газ подвергается воздействию нейтрально заряженных атомов и высвобождает отрицательно заряженные электроны, а сам ионизируется - превращается в положительные ионы. Они разгоняются под воздействием электростатического поля и, вылетая из двигателя, придают спутнику необходимый импульс".
Аноним 19/01/23 Чтв 07:18:50 740640 357
>>740639
Может уже скажешь, что сказать хотел? Тебя вот эта часть смущает?
>Газ подвергается воздействию нейтрально заряженных атомов
Аноним 19/01/23 Чтв 08:39:31 740644 358
>>740619
>это просто такая рандомная константа, выпавшая нашей вселенной?
Именно так. Для частиц типа фотона это просто одно из свойств. Гипотетически могли бы быть похожие частицы, но имеющие скорость, превышающую скорость света в вакууме, - тахионы. Но таких частиц не существует. А для массивной частицы невозможно достичь даже скорости света по той причине, что для этого потребуется бесконечная энергия. Бесконечную энергию брать неоткуда. Поэтому скорость света в вакууме - это фундаментальный предел, который можно обойти лишь через манипуляции с кривизной пространства (варп-технологии).
Аноним 19/01/23 Чтв 09:24:32 740646 359
>>740644
> Для частиц типа фотона
Разве фотон это частица, а не квант поля? Или это одно и тоже?
Аноним 19/01/23 Чтв 10:20:17 740648 360
Аноним 19/01/23 Чтв 10:21:36 740650 361
>>740644
>Гипотетически могли бы быть похожие частицы, но имеющие скорость, превышающую скорость света в вакууме, - тахионы. Но таких частиц не существует
Их пока не нашли, пофиксил тебя.
Аноним 19/01/23 Чтв 11:03:51 740651 362
>>740648
Ну типо да, а то ты косвенно признал плотноприжатые фотоны
Аноним 19/01/23 Чтв 13:08:43 740659 363
>>740644
Да, весьма странненько конечно это, при таких массштабах вселенной такое ограничение.
Аноним 19/01/23 Чтв 13:13:04 740660 364
>>740644
>А для массивной частицы невозможно достичь даже скорости света
Ну кстати почти возможно, всякие частицы бога достигают вроде как 99,(9)%, но они видимо только в квазарах/блазарах, где действительно ебанутые совершенно энергии.
Аноним 19/01/23 Чтв 23:22:17 740746 365
Buoyancy.svg.png 83Кб, 800x999
800x999
Будут ли пузыри в подлёдном океане Европы подниматься быстрее или медленней? Я просто не могу понять - там же вроде гравитация ниже, так что они должны типа быть медленней подниматься, или же наоборот?
Аноним 20/01/23 Птн 01:06:32 740750 366
>>740746
выталкивание тела с воды определяется законом Архимеда, что в свою, очередь равен произведению плотности предмета, высоты и ускорению свободного падения. известно, что самое высокое ускорение свободного падения на экваторе, самое низкое у полюсов. соответственно сила Архимеда на полюсах будет ниже, скорость всплывания также понизиться
Аноним 20/01/23 Птн 04:31:38 740754 367
>>740746
В таких случаях бывает полезно представить, что происходит в предельном случае. Возьми нулевую гравитацию. Куда пузырь поплывет? Понятно, никуда, значит скорость нулевая будет. Значит чем меньше гравитация, тем меньше скорость.

Это, конечно, ещё очевидно из силы Архимеда, но не для всех ситуаций есть формулы.
Аноним 20/01/23 Птн 07:42:07 740760 368
>>740646
Одно и то же.
>>740650
Их не существует.
>>740660
Они не достигают скорости света ни при каких условиях. "Почти" - это не скорость света. Частицам в ультрарелятивистских джетах не хватает долей процента до скорости света. Они эти доли не смогут добрать никогда, поскольку для этого нужна бесконечная энергия.
Аноним 20/01/23 Птн 08:42:20 740762 369
>>740659
Ничего странного. В масштабах мультиверса существуют вселенные с разными наборами фундаментальных констант. Существуют вселенные, в которых скорость света имеет иное значение. Правда, в таких вселенных невозможна никакая жизнь. В нашей Вселенной мы и существуем благодаря тому, что константы, включая скорость света, имеют такие значения, какие имеют - они благоприятствуют существованию разумной жизни.
Аноним 20/01/23 Птн 11:45:41 740786 370
>>740762
а еще есть вселенная где 8 измерений, сам лично видел, врать не буду.
Аноним 20/01/23 Птн 13:04:27 740797 371
>>740786
Антисетипетух, спокойно.
Прецессия северного полюса Земли fat_alien 20/01/23 Птн 20:34:13 740825 372
Прецессиясеверн[...].png 521Кб, 1136x1136
1136x1136
Npolesqare30deg[...].png 595Кб, 1136x1136
1136x1136
Уникальное изображение для 2ч. такого нет во всём интернете! Доказательство — https://en.wikipedia.org/wiki/Axial_precession

Проекция пути северного полюса Земли на неподвижное небо эпохи J2000.0 при лунно-солнечной и планетной[1] прецессии для интервала времени от 48000 г. до н. э. до 52000 г. н. э. В центре показан путь прецессии северного полюса орбиты Земли для того же времени и белым крестом обозначено направление северного полюса инвариантной плоскости[2]. Голубой сеткой обозначена эклиптическая система координат и жёлтой — экваториальная система координат для эпохи J2000.0. Для собственного вращения Земли зависимости ψA и ωA, для эклиптики — PA и QA из статьи [3]

1. Бакулин П.И. Курс общей астрономии. — 4-е изд.. — М.: «Наука», 1977. — 544 с.
2. D. Souami, J. Souchay. The solar system’s invariable plane (англ.) // Astronomy & Astrophysics. — 2012-07-01. — Vol. 543. — P. A133. — ISSN 1432-0746 0004-6361, 1432-0746. — doi:10.1051/0004-6361/201219011.
3. J. Vondrák, N. Capitaine, P. Wallace. New precession expressions, valid for long time intervals (англ.) // Astronomy & Astrophysics. — 2011-10-01. — Vol. 534. — P. A22. — ISSN 1432-0746 0004-6361, 1432-0746. — doi:10.1051/0004-6361/201117274.
Аноним 20/01/23 Птн 20:57:58 740827 373
>>740825
Актуальная инфа, я как раз задавался вопросом, как я буду ориентироваться по звездам спустя двадцать тысяч лет, когда человечество скатится в каменный век.
Аноним 20/01/23 Птн 22:10:38 740830 374
image.png 1984Кб, 1226x911
1226x911
Почему ещё не запустили межзвездный зонд на солнечном парусе? Один жалкий микрозапуск был на НОО, причем какими-то бичами.
Почему НАСА не хочет серьезно испытать технологию и узнать, до какой скорости сможет разогнаться парусник дополнительно с использованием грави-маневров?
Аноним 20/01/23 Птн 22:17:58 740831 375
>>740830
Уважаемые инопланетные партнёры не разрешают.
Аноним 20/01/23 Птн 22:29:09 740832 376
>>740830
1. Нетзодач.
2. Нетденех.
3. Он разгоняться дольше будет чем электроника проживет.
4. Нетзодач.
5. ???????
6. Нетденех.
Аноним 20/01/23 Птн 23:52:51 740833 377
можно ли замедлить вращение Земли, если врыть в землю 10 тыс ракетных двигателей и запустить их все разом чтобы они толкали планету в противоположное направление?
Аноним 21/01/23 Суб 00:04:13 740834 378
vmnc1lrHw10.jpg 72Кб, 567x604
567x604
>>740830
>межзвездный зонд на солнечном парусе
Пролетающие мимопланетяне на безопорных кефирогравицапах засмеют же.
Аноним 21/01/23 Суб 06:45:31 740844 379
>>740833
Нет, если все продукты горения двигателей будут оставаться на Земле.
Аноним 21/01/23 Суб 10:02:42 740846 380
lsst1.jpg 895Кб, 1536x994
1536x994
изображение.png 2510Кб, 1600x1068
1600x1068
изображение.png 1127Кб, 1280x958
1280x958
изображение.png 2122Кб, 1600x1097
1600x1097
>>738638 (OP)
>Тред Тупых Вопросов
О, как раз для меня. Вот у телескопа LSST (им. Веры Рубин) фотокамера на 3,2 млрд пикселей и объектив на 1,5 метра. И это самая большая такая система в телескопах, как я понял, а у других телескопов какие системы раньше стояли? Имею ввиду, сколько пикселей было?
Аноним 21/01/23 Суб 10:17:05 740847 381
LSST Paul-Baker.jpg 115Кб, 720x556
720x556
как то так он должен работать
Аноним 21/01/23 Суб 10:53:07 740849 382
что будет если гипотетически прорыть туннель сквозь Землю и прыгнуть в него? долетишь до центра а потом? Там остановишься?
Аноним 21/01/23 Суб 11:14:08 740850 383
>>740849
Это же просто, сам подумай. Ты когда падал, ускорялся, чтобы остановиться у тебя должно быть ускорение с обратным знаком. Так с хуя ли оно у тебя будет там где гравитация станет около нуля при уравновешивании массы со всех сторон? Ты просто дальше полетишь на другую сторону и тогда гравитация уже будет отнимать у тебя ускорение, до поверхности не долетишь, т.к. часть ускорения была потеряна на трение о воздух. Так и будешь болтаться как маятник пока не растеряешь все ускорение и не остановишься в центре.
Аноним 21/01/23 Суб 12:05:25 740852 384
>>740850
хорошо. А как человек будет ощущать себя в центре? как в невесомости?
Аноним 21/01/23 Суб 12:27:44 740854 385
>>740852
Невесомость это состояние падения.
Аноним 21/01/23 Суб 13:21:50 740859 386
>>740830
>хочет серьезно испытать технологию и узнать, до какой скорости сможет разогнаться парусник дополнительно с использованием грави-маневров
Зачем строить, когда можно посчитать?
Аноним 21/01/23 Суб 13:29:46 740861 387
>>740832
>1. Нетзодач.
Есть, исследование солнечной системы и межзвездной среды.
>2. Нетденех.
Есть, лол.
>3. Он разгоняться дольше будет чем электроника проживет.
С хуя ли? На НОО он держался долго, учитывая, что его постоянно тормозила атмосфера. Если далеко от планет, то атмосферы нет и он будет разгоняться всё время полета.
>4. Нетзодач.
Есть.
>5. ???????
>6. Нетденех.
Есть.
Аноним 21/01/23 Суб 13:34:04 740862 388
>>740859
>Зачем строить, когда можно посчитать?
Пока быдло дрочило циферки и хрюкало о экономической нецелесообразности полета на марс, илон маск просто и без задней мысли строит ракету для полета на марс, а теперь все им восхищаются и пытаются повторить.
История всегда показывала, что эмпирический опыт и итерационный подход > дрочки циферок, но пидорахи всё живут 19 веком и дрочат циферки.
Аноним 21/01/23 Суб 13:35:24 740863 389
>>740861
>Есть
По твоему НАСА берет деньги в тумбочке, а не выбивает их из конгресса, доказывая что вот именно этот лишний миллиард ответит на вопрос жизни, вселенной и всего такого?
Аноним 21/01/23 Суб 13:41:31 740864 390
>>740852
Да, хотя свободное падение это и есть невесомость, правда ускорение даёт перегрузку.
Аноним 21/01/23 Суб 14:15:28 740867 391
>>740862
Как раз проебывать кучу средств на постройку йоб с целью посмотреть что будет это пидораший подход, а белые люди в циферки могут.
Аноним 21/01/23 Суб 14:25:12 740868 392
>>740864
>свободное падение это и есть невесомость
Масоны говорят что во всей Вселенной нет места где бы любое тело находилось бы в покое а не падало куда-то.
Аноним 21/01/23 Суб 16:01:19 740879 393
>>740868
Ну да, все в движении. Но ускорение мы вполне ощущаем.
Аноним 21/01/23 Суб 16:34:35 740880 394
>>740868
Када силы действует одновременно равно на все частицы твоего тела, то ты не чувствуешь ускорения - невесомость. Када силы действуют по разному на частички тела, то испытываешь дискомфорт - одна часть тела ускоряется быстрее другой и получается тянущее ощущение. Када сила действует через опору на часть твоего тела, то уже прям конкретно чувствуешь эту силу.
Аноним 21/01/23 Суб 18:05:12 740881 395
>>740854
только без ветра
Аноним 21/01/23 Суб 19:44:04 740884 396
существует ли неизученное излучение? Например с длиной волны меньше чем у гамма?
Аноним 21/01/23 Суб 23:14:29 740893 397
Аноним 21/01/23 Суб 23:15:51 740895 398
>>740884
> существует ли неизученное излучение?
Ближайшее: Хокинга.
Аноним 22/01/23 Вск 01:57:20 740896 399
Нахуя тратить сотни миллиардов на бесполезные телескопы, когда можно вложить их и умы их изобретающие в развитие двигательных установок, что даст куда больше выхлопа в будущем.
Аноним 22/01/23 Вск 07:17:32 740897 400
>>740896
Химические двигатели подошли к своему пределу, все самые энергетически эффективные реакции уже испытаны.
Электрические двигатели дают мизерную тягу и для пилотируемых полетов бесполезны.
Паруса дают мизерную тягу и для пилотируемых полетов бесполезны.
Ядерные двигатели для старта с земли бесполезны, а для полетов внутри солнечной системы малоэффективны так как требуют рабочего тела.
Остаются термоядерные двигатели, но управляемая термоядерная реакции еще на Земле не освоена, прежде чем конструировать на ней двигатель для космоса. Да и рабочее тело для них так же нужно.
Все уже давно исследовано и подсчитано.
Аноним 22/01/23 Вск 07:36:13 740898 401
>>740895
У излучения Хокинга длина волны не меньше, чем у гамма. Это обычное электромагнитное излучение.
>>740884
Весь электромагнитный спектр открыли еще в начале прошлого века.
Аноним 22/01/23 Вск 07:46:17 740899 402
>>740862
>илон маск просто и без задней мысли строит ракету для полета на марс
Ну и где полёт на Марс? Опять отложен до две тысячи такого-то года?
Аноним 22/01/23 Вск 09:00:57 740900 403
>>740898
>У излучения Хокинга длина волны
Я что - то пропустил? Его уже экспериментально обнаружили?
Аноним 22/01/23 Вск 09:01:24 740901 404
>>740897
>Химические двигатели подошли к своему пределу, все самые энергетически эффективные реакции уже испытаны.
Когда завезут кубический гош-азот, тогда и поговорим.
>Электрические двигатели дают мизерную тягу и для пилотируемых полетов бесполезны.
Когда завезут реактор на вторичных электронах с 10-100МВт электрической мощности, тогда и поговорим.
>Паруса дают мизерную тягу и для пилотируемых полетов бесполезны.
Когда завезут разгонный лазер, тогда и поговорим.
>Ядерные двигатели для старта с земли бесполезны, а для полетов внутри солнечной системы малоэффективны так как требуют рабочего тела.
Когда завезут ГФЯРД, тогда и поговорим.
>Остаются термоядерные двигатели, но управляемая термоядерная реакции еще на Земле не освоена, прежде чем конструировать на ней двигатель для космоса. Да и рабочее тело для них так же нужно.
Когда завезут импульсный высокочастотный Z-пинч, тогда и поговорим.
Аноним 22/01/23 Вск 10:54:51 740904 405
>>740901
>Когда завезут кубический гош-азот
Все возможные химические реакции давно перепробованы, так что увы.
По остальным пунктам даже лень писать.
Аноним 22/01/23 Вск 12:21:58 740907 406
>>740901
Не знаю, что за кубический азот, но металлический водород тема.
Аноним 22/01/23 Вск 14:24:48 740918 407
>>740762
> Ничего странного. В масштабах мультиверса существуют вселенные с разными наборами фундаментальных констант. Существуют вселенные, в которых скорость света имеет иное значение. Правда, в таких вселенных невозможна никакая жизнь. В нашей Вселенной мы и существуем благодаря тому, что константы, включая скорость света, имеют такие значения, какие имеют - они благоприятствуют существованию разумной жизни.
большей научпокерной хуйни не читал
Аноним 22/01/23 Вск 15:16:37 740922 408
>>740918
Все правильно он написал, иди в /зог/ воюй с правдой, шизло.
Аноним 22/01/23 Вск 18:48:09 740936 409
>>738638 (OP)
Существуют ли инопланетяне?
И если да, то почему мы их не видим.
Аноним 22/01/23 Вск 19:26:16 740937 410
>>740936
Потому что не положено.
Аноним 22/01/23 Вск 19:48:18 740938 411
>>740937
Матрица чтоли? Текстуры ограничены? Давайте попиздим об этом, интересно же.
Аноним 22/01/23 Вск 20:19:45 740940 412
>>740936
>Существуют ли инопланетяне?
Наверное.
>И если да, то почему мы их не видим.
Далеко.
Аноним 22/01/23 Вск 20:59:40 740943 413
>>740896
нихуя это не даст, лучше вложится в исследование 2д тян и девочек-лисичек
Аноним 23/01/23 Пнд 00:48:22 740951 414
>>740897
Мда, глупость какоя-то, неужели так сложно просто грести веслами в эфире.
Аноним 23/01/23 Пнд 06:30:00 740954 415
джва кефира.jpg 91Кб, 1000x710
1000x710
>>740951
Удваиваю. Заколебали уже на пердячей тяге летать, как диды. Давайте разрабатываете что нибудь новое, молодежное.
Аноним 23/01/23 Пнд 07:02:57 740958 416
>>740936
>И если да, то почему мы их не видим.
На это выдумали миллиард объяснений и объединили под парадоксом Ферми. Ознакомься и выбирай какое тебе ближе.
Аноним 23/01/23 Пнд 07:05:21 740959 417
>>740943
>исследование 2д тян
Чё ты в них исследовать собрался?
Аноним 23/01/23 Пнд 08:03:57 740961 418
>>739049
Скорость выше из-за большего bandwidth, а не частоты. У 2,4 ГГц вифи реалистично в многоквартирном доме будет 20 МГц bandwidth, а в вифи 7 грядущем - в 6 ГГц можно получить ширину канала 320 МГц - скорость будет только за счет этого (а там еще много чего помимо) в 16 раз больше
Аноним 23/01/23 Пнд 11:05:41 740962 419
>>740904
>Все возможные химические реакции давно перепробованы
Азотистые и борные цепочки, метастабильные соединения, образующиеся при сверхвысоких давлениях, только начали изучаться. Есть ещё гипотезы об экзотической сверхвалентной химии с задействованием внутренних электронов, ядерно-химических реакций с дестабилизацией ядра полной ионизацией всех оболочек, схемы по стабилизации озона и.т.п.
Аноним 23/01/23 Пнд 11:33:47 740965 420
>>740918
плюсую. Какие то фантазии свои расписал.
Аноним 23/01/23 Пнд 13:46:26 740970 421
Аноним 23/01/23 Пнд 17:06:05 740979 422
>>740959
не исследовать, а создавать и продавать двачерам
Аноним 23/01/23 Пнд 17:07:25 740980 423
>>740900
Я про модель.
>>740918
Антисетипетух, не трясись так.
>>740936
>Существуют ли инопланетяне?
Да.
>почему мы их не видим
Видим.
Аноним 23/01/23 Пнд 17:10:17 740981 424
>>740979
Так хуйдожники и так их создают и продают. А если ты про то, чтобы материализовать их, то это будет уже не 2д, а 3д.
Аноним 23/01/23 Пнд 17:23:42 740986 425
>>740981
2д тяны могут существовать ирл, одномерные струны же существуют
Аноним 23/01/23 Пнд 22:58:31 740995 426
Без названия.jpeg 13Кб, 202x250
202x250
>>738638 (OP)
Аноны, приготовьтесь. Сейчас будет ОЧЕНЬ ТУПОЙ ВОПРОС.

Пишу фантастическую книгу. Конечно же, в жанре "твердой" научной фантастики, но с элементами космооперы.
Книга рассказывает о войне будущего между землянами и колонистами-повстанцами, происходящей на одной из планет Солнечной системы. Много внимания уделено именно воздушным боям (авиация). В связи с чем у меня возникает вопрос:
КАКУЮ ИМЕННО ПЛАНЕТУ ВЫБРАТЬ?
МАРС? ВЕНЕРУ? ТИТАН?
Аноним 23/01/23 Пнд 23:26:36 740996 427
>>740995
Марс точно нет, атмосфера там очень неплотная
Титан хуй знает, из-за относительно высокой плотности атмосферы и низкой гравитации там будет летать что угодно и оптимальный летательный аппарат будет выглядеть очень нелепо если не путаю, крылья там нужны в раз 10 меньше. Любое падение можно предотвратить крошечным парашютом, в том числе для всего аппарата.
Венера заебись. За счет очень разной плотности на разных высотах можно придумать дохуя ситуаций и конструкций аппаратов. На самом низу огромное давление и температура, можно обыграть разные тактические приемы и так далее, + опасность при потере контроля над аппаратом. Всякие дирижабли и прочее в средних слоях, много чего.
Аноним 24/01/23 Втр 00:12:26 740998 428
image.png 216Кб, 800x765
800x765
Какую частицу предсказали следующей, что будут проверять и находить после Хиггса?
Аноним 24/01/23 Втр 01:03:08 741000 429
>>740998
Ну есть всякие там гравитоны-инфлатоны, но чтобы их сделать - нужен коллайдер размером с экватор как минимум.
Аноним 24/01/23 Втр 02:25:46 741005 430
>>740995
Расклад такой - за Марсом энергии мало, так что отраженный от Венеры свет можно фокусировать и отправлять лазером или ещё как-то замарсовым бомжам. Т.е. не дают им энергии и все, им пизда, поэтому они будут в рабстве у землян и венерианцев, это очевидно. Марсобомжи в промежуточном положении, но все равно ни атмосферы, ничего, лишенцы. Вся система во власти земле-венерианской межпланетной конфедерации. Ну а потом когда до сферы Дайсона дойдет, когда все выгребут за Нептуном, все равно опять же все собирутся до орбиты Марса, смысла нет сферу растягивать.
Аноним 24/01/23 Втр 02:45:31 741008 431
>>740995
дохуя твёрдые колонисты или марс или меркурий(безвоздушная авиация, почему бы нет), для титана и галилеевых уже надо прикручивать масс термояд, но тоже довольно твёрдо, венера максимум сайфай
Аноним 24/01/23 Втр 02:57:49 741010 432
>>740998
никакую, стандартная модель закрыта, ищут даркматер, суперсимметрию и вообще любые расхождения, дальше только пространственная структура и микро чд, но на около сегодняшних или технологиях ближайшего времени для них нужны коллайдеры минимум соответственно размером с солнечную систему и 1/10 галактики
Аноним 24/01/23 Втр 09:02:07 741018 433
>>741008
>венера максимум сайфай
Почему?
Аноним 24/01/23 Втр 09:10:43 741020 434
>>741018
пятьсот цельсиёв на поверхности, для колонизации нужны всякие астроинженерные зонтики соизмеримые по размеру с самой планетой и сотни лет на остывание поверхности
Аноним 24/01/23 Втр 09:39:38 741021 435
>>741020
Нет. Нужны всего лишь:
1. Термояд.
2. Здоровенные дирижабли (с воздухом).
3. Роботы, способные добывать полезные ископаемые с поверзности самой планеты.
4. Хорошие скафандры.
5. Медведи Нанотрубки, миллиарды нанотрубок!

Все это вполне осуществимо к 24-25 веку.
Аноним 24/01/23 Втр 10:07:36 741022 436
image.png 336Кб, 732x549
732x549
Какое кровяное давление у астронавтов было которые летали в кораблях с 0.3 атмосферы кислорода? Они же меряли себе там давления?
Если корабли надувать на 0.3 вместо одной атмосферы воздуха, то какие профиты?
Аноним 24/01/23 Втр 10:27:01 741023 437
>>741021
> Роботы, способные добывать полезные ископаемые с поверзности самой планеты.
увы, бездушный сопромат не разрешает копать при 500 градусах (и сотне атмосфер) больше чайной ложки в год, с последующей дефектовкой и пересборкой всего
> 1. Термояд.
нечем охлаждать
> 4. Хорошие скафандры.
тоже увы, и в тени в вакууме имеют ограниченный ресурс охлаждения, даже если добавить сотню-другую лет - 500 градусов не осилить, нет таких материалов
> 2. Здоровенные дирижабли (с воздухом).
а в чем смысл колонизировать воздух? на высотах где температура не ебёт - и тяги как на земле мизер, а ветра ебут
это буквально самое адище в солнечной системе, хуже только газовые гиганты, скорее реально зонтик замутят чем там будет что-то больше исследовательской станции
Аноним 24/01/23 Втр 10:32:33 741025 438
Шлюшня.png 585Кб, 453x604
453x604
>>740995
Отрывок скинь хоть, засранец, я может картинку в книгу нарисую
Аноним 24/01/23 Втр 11:00:53 741026 439
>>741023
>увы, бездушный сопромат не разрешает копать при 500 градусах (и сотне атмосфер) больше чайной ложки в год, с последующей дефектовкой и пересборкой всего

Сопромат 21 века.
А, скажем, века 26-го?..
В 18 веке люди и подумать не могли о добыче полезных ископаемых из океана, а уже в 2020 году был разработан прототип робота для этих целей. Всего 300 лет - и такой прогресс!

>нечем охлаждать
Разберемся. Как вариант: литий.

>500 градусов не осилить, нет таких материалов
А нам и не нужно 500.
Действие происходит на высоте 45-60 км. Температура: от 60°С до 30°С. Проблема только в клятой серной кислоте и углекислом газе.

>а в чем смысл колонизировать воздух? на высотах где температура не ебёт - и тяги как на земле мизер, а ветра ебут

Во-первых, ЯСКОЗАЛ.
Во-вторых, дейтерий и тритий.
В-третьих, оптимальная для жизни и размножения сила тяжести (нигде больше вы такого не найдете).
В-четвертых, пруфы ветров и "тяги" (ты хотел сказать что-то другое)?

>солнечный зонтик
Впрочем, можно.
Аноним 24/01/23 Втр 11:02:34 741027 440
>>741025
Я написал 1-2 главы. Пока нечего кидать.
Аноним 24/01/23 Втр 11:02:51 741028 441
>>741022
Все профиты какие есть. Только не 0,3 а 0,2. А азотом дышат только земледебилы с негнущимися скафандрами, эти земледебилы напроектировали лунный сьют и потом их нилы армстронги хуями крыли потому что давление в скафандре блять должно быть низким!
Аноним 24/01/23 Втр 11:07:28 741029 442
>>741026
База, фундамент. Просто скажите: "Венера круто!"
Аноним 24/01/23 Втр 11:30:18 741034 443
Аноним 24/01/23 Втр 15:23:40 741093 444
>>741021
>>741023
Вроде на Венере на возвышенностях более-менее температура и давление, не?
Аноним 24/01/23 Втр 15:35:38 741098 445
>>741093
Вот-вот! А значит:
1. Колонисты живут в парящих в небесах городах на высоте 50 км, добывают воду из кислоты, а кислород - из СО2, выращивают яблоки и персики, просто и без задней мысли.
2. Внизу копашаться роботы-харвестеры, добывающие для них ресурсы прямо из планетарной коры. Ресурсы упаковываются и поднимаются вверх при помощи воздушного шара с водородом.
3. Роботы-курьеры ловят наши "ресурспаки" и доставляют на заводы на высоте ~40 км.
4. Заводы перерабатывают ресурсы, штампуют новые небесные города и новых роботов: курьеров выпускают, а харвестеры просто бросабт вниз (когда предыдущая смена "дохнет").
5. Бонусом идут несколько орбитальных "затенителей", понижающих температуру в месте работы харвестеров до 400~350°С.
6.???
7. ПРОФИТ

Только одна ремарочка...
Все это будет не раньше 2400 года.
Аноним 24/01/23 Втр 15:41:04 741100 446
>>741098
Проблема в том что эта система выглядит настолько неэффективной и сложной во всех компонентах, что нужно очень сильное обоснование нахуй колонии вообще нужны там.
Аноним 24/01/23 Втр 15:56:59 741107 447
>>741100
Зачем там колония?
Интересный вопрос. Вижу 3 варианта:

1. ЯСКОЗАЛ.

2. Человечество так и не изобрело способ выживать при низкой силе тяжести, ГМО запрещено, а людишек на Земле все больше.

3. Какой-то эксцентричный миллиардер в 2220 году говорит: "ХАЧУ ВЕНЕРУ" и начинает строительство всей этой балды.

4. Прилетают злобные ксеносы, которые хотят выпилить людишек. Единственный способ пережить их нашествие - спрятаться там, куда они не полезут, а именно у негра в жопе в густом мутном сиропе венерианской атмосферы.

Прошу прощения за ошибки, я житель Страны Вина, русский знаю плохо.
Аноним 24/01/23 Втр 15:57:27 741108 448
>>741100
Зачем там колония?
Интересный вопрос. Вижу 4 варианта:

1. ЯСКОЗАЛ.

2. Человечество так и не изобрело способ выживать при низкой силе тяжести, ГМО запрещено, а людишек на Земле все больше.

3. Какой-то эксцентричный миллиардер в 2220 году говорит: "ХАЧУ ВЕНЕРУ" и начинает строительство всей этой балды.

4. Прилетают злобные ксеносы, которые хотят выпилить людишек. Единственный способ пережить их нашествие - спрятаться там, куда они не полезут, а именно у негра в жопе в густом мутном сиропе венерианской атмосферы.

Прошу прощения за ошибки, я житель Страны Вина, русский знаю плохо.
Аноним 24/01/23 Втр 16:05:23 741112 449
>>741108
>>741107
Но эти причины выглядят крайне притянутыми, а у тебя вроде как твердая фантастика. Именно в этом проблема, я считаю. Титан лучше.
Аноним 24/01/23 Втр 16:11:13 741116 450
>>741112
>у тебя вроде как твердая фантастика
С элементами космооперы жи, дон.
Аноним 24/01/23 Втр 16:24:16 741119 451
>>741112
Титан, Титан. Там дебилы живут! Вы поймите я всю систему объездил, это не землянин, не марсианин, не венерианин. Ты смотришь на него вот такой... Просто титанская термическая аномалия, мозги себе все отморозили.
Аноним 24/01/23 Втр 16:30:02 741121 452
>>741119
Проблема в том, что до Титана лететь минимум 18 месяцев (с термоядом), IRL - 6 лет. Вести войну с "отморозками" в таких условиях невозможно. Просто невыгодно да и незачем.
А до Венеры с термоядом жалких 3-4 месяца. Плюс там есть за что воевать: дейтерий-тритиевые заводы, система управления орбитальными отражптелями... Без всего этого о Юпитере и Сатурне можно не думать.
Аноним 24/01/23 Втр 16:36:44 741127 453
Победа? Аноним 24/01/23 Втр 17:34:23 741140 454
Венериане победили в бою за этот тред?
Аноним 24/01/23 Втр 17:51:10 741145 455
image.png 101Кб, 498x379
498x379
Аноним 24/01/23 Втр 18:52:41 741152 456
>>740998
Никакую. Стандартная модель с открытием бозона Хиггса завершена. Сейчас в физике элементарных частиц настали времена поиска того, не знаю чего. Суперсимметрию ищут - нихуя не находят, ловят частицы темного вещества - нихуя не могут поймать. Все гоняются за новой физикой, а где вылезет эта новая физика - никто не знает.
Аноним 24/01/23 Втр 18:54:12 741153 457
>>741152
>Стандартная модель с открытием бозона Хиггса завершена.
Что-то мне кажется, что и до Хиггса такое говорили, и при этом о преонах говорили.
>Суперсимметрию ищут
БАК ее не опроверг разве?
Аноним 24/01/23 Втр 18:55:33 741154 458
image.png 144Кб, 629x640
629x640
>>741121
>>741127
Нет. Смотрите скорость выхлопа - с такими цифрами без проблем можно разогнаться до примерно такой же скорости и даже повыше. До Сатурна полтора световых часа, даже в худшем случае это будет не больше чем несколько дней.
Аноним 24/01/23 Втр 19:01:38 741157 459
>>741153
>Что-то мне кажется, что и до Хиггса такое говорили
Не говорили, тебе лишь кажется.
>о преонах говорили
В контексте серьезной новой физики, которую нужно искать, - нет, не говорили. Сейчас о них тем более не говорят, учитывая, что множество преонных моделей просто закрываются экспериментальными данными.
>БАК ее не опроверг разве?
Всего лишь не обнаружил наличие суперпартнеров на текущей доступной энергии. Наиболее популярные модели суперсимметрии это закрыло, но не закрыло саму суперсимметрию.
Аноним 24/01/23 Втр 19:26:01 741160 460
>>741154
> лететь со скоростью выхлопа
ты выбрал верный тред чтобы об этом написать
Аноним 24/01/23 Втр 19:46:51 741167 461
Аноним 24/01/23 Втр 20:04:33 741170 462
>>741160
Какие проблемы? Нынешние химические ракеты как раз летают со скоростью выхлопа или около, если смотреть на отдельную ступень.
Аноним 24/01/23 Втр 21:18:21 741177 463
Если человек порежется или поцарапается на Земле, кровь начнёт сворачиваться и рана будет заживать.
А если на каком-нибудь Марсе в местное лето, в жаркий (+35 по Цельсию - зарегистрированный рекорд, ну пусть будут +25) день, мимопроходил рандомный обычный стандартный земной человек радиацию не учитываем в шлеме, но без скафандра взял, споткнулся и упал на местные камни, порезавшись. Будет ли рана заживать? Ведь кислорода в атмосфере нет, азота нет, значит, крови не обо что загустевать. А если мы услими атмосферу Марса до 1/3 плотности атмосферы Земли?
А если то же самое, но в вакууме открытого космоса?
Аноним 24/01/23 Втр 21:20:23 741179 464
>>741170
Послушай, землянин, в любом случае даже термояду не разогнаться до такой скорости, чтоб долететь до Титана хотя бы за полгода. А вот Венера - цель вполне достижимая, хоть и не самая уютная. Умные дядьки уже все продумали: как колонизировать Венеру, как добывать ресурсы, как жить и выращивать там апельсины. А Титан, как по мне, слишком заежджен.
В любом случае: я уже сделал выбор. Как молодой писатель-фантаст (дебильный, конечно, но уже популярный в нккоторых кругах) я ставлю перед собой цель НАСЕЛИТЬ ВЕНЕРУ ЛЮДЬМИ, УСТРОИТЬ ВОЙНУ С ЗЕМЛЕЙ И НАКОНЕЦ ДАТЬ ЭТОЙ ПЛАНЕТЕ НЕЗАВИСИМОСТЬ.
А вы как хотите.
Аноним 24/01/23 Втр 22:44:25 741193 465
>>741177
Ты кровь с цианоакрилатом спутал? Он тоже не от кислорода густеет, кстати.
Почитай от чего и как кровь сворачивается.
Аноним 25/01/23 Срд 04:38:29 741216 466
16746000909430-[...].png 476Кб, 1039x815
1039x815
Мнение?
Аноним 25/01/23 Срд 07:22:20 741218 467
>>741216
Читал иную версию новости: мол, в этом, 2023 году, обнаружили, что ядро Земли остановилось и стало вращаться в обратную сторону аж в 2009 году
Аноним 25/01/23 Срд 09:55:03 741219 468
>>741177
ты молодой шутливый не вскрывай эту тему
Аноним 25/01/23 Срд 10:00:25 741220 469
>>741216
что же заставит милионы тонн ёбаного расплава остановиться , а потом хуяк, и заставит вращаться в другую блять сторону? гендальф штоли? или говарт лавкрафт ?
Аноним 25/01/23 Срд 10:21:29 741221 470
>>741216
Скидывай научную публикацию, а не скрин желтушной хуйни.
Аноним 25/01/23 Срд 10:44:28 741222 471
>>741220
Плотноприжатые фотоны Магнетизм. Он ещё и космические лучи умеет останавливать.
Аноним 25/01/23 Срд 13:38:06 741235 472
>>741222
так вращение ядра создаёт магнитное поле, ёптыбля, причина и следствие
Аноним 25/01/23 Срд 17:08:28 741264 473
>>741235
Магнитное поле создается не вращением ядра, а конвекции проводящего вещества в мантии и на границе внешнего ядра с мантией.
Аноним 25/01/23 Срд 18:02:20 741268 474
image 1211Кб, 1083x1080
1083x1080
Часто видел упоминание Сфер Дайсона. Допустим, мы спустя миллион лет захотим построить Сферу Дайсона. Где, блять, мы возьмём столько ресурсов? Как мы всё это будем строить? Звезда же чудовищных размеров. Кажется, что Сфера Дайсона это совсем уж какая-то нереальная маня-фантазия, типа Варп Двигателя.
Аноним 25/01/23 Срд 18:56:31 741270 475
>>741268
>мы спустя миллион лет захотим
Не захотим, от человечества скорее всего к тому времени останется только пыль.

>Где, блять, мы возьмём столько ресурсов?
Ну если вдруг мы действительно доживем и ахуеть как разовьемся, то есть мнение, что нами будут изобретены куда более ахуительные и компактные элементы питания и сфера будет попросту не интересна. Да и ничто не мешает высосать досуха необитаемую планету.

>Как мы всё это будем строить? Звезда же чудовищных размеров.
Выкатываешь на орбиту хуилион дронов, которые конвеером еще будут таскать ресурсы с ближайшей планетки и перерабатывая строить сферу. Понятное дело, займет это пиздец как много времени, но если уж мы миллион лет прожили, то и сферу достроим.

Ну а если честно, то хуйня без задач.
Аноним 25/01/23 Срд 18:59:09 741271 476
Аноним 25/01/23 Срд 20:18:42 741290 477
>>741271
есть бляТь, и есть бляДь
Аноним 25/01/23 Срд 20:59:34 741305 478
>>741290
Да. Первое - неправильно, второе - правильно.
Аноним 26/01/23 Чтв 06:55:12 741347 479
>>741268
Сфера хуйня, как ни крути. А вот Рой Дайсона из кучи спутников с солнечными панелями уже звучит реалистичней, и возможен даже с сегодняшними технологиями, хоть завтра можно запускать. Но без какого-то способа удаленной доставки энергии смысла в этом нет.
Аноним 26/01/23 Чтв 09:09:25 741353 480
>>741268
>типа Варп Двигателя
Что такого "нереального" в варп-двигателе, долбоеб?
Аноним 26/01/23 Чтв 11:18:33 741366 481
>>741353
То что его даже на бумаге не существует, долбоёб.
Аноним 26/01/23 Чтв 16:44:54 741401 482
>>741366
Теоретически он возможен же. Надо только научиться управлять гравитационным полем.
Аноним 26/01/23 Чтв 17:14:04 741403 483
>>741401
Именно технически он не возможен. Нет способов манипуляцией с гравитацией кроме собирание большего количества массы в малом пространстве, что самом по себе не даст собрать варп-драйв.
Аноним 26/01/23 Чтв 17:19:27 741404 484
>>741403
Ему кроме того нужна "отрицательная материя" которой нет нигде кроме засранных мозгов пузырьдебилов. Блять ДАЖЕ в альтернативных моделях теоретической физики, которых нахуй сотни и тысячи, нет такой хуйни! Пузырьдебил хуже антисетипитуха нахуй.
Аноним 26/01/23 Чтв 17:31:43 741405 485
>>741268
Проапгрейжена сфера Джайсона в виде многослойного роя вполне реальны. Много материала не надо, ведь считается по перекрываемой площади, а не по объему. Ресурсы можно брать с Меркурия, которого хватит не только на постройку и последующий ремонт, но еще на строительство инфраструктуры переноса энергии аж до Нептуна.
Вообще это мегасооружение чрезвычайно заманчивая штука, ибо дает громадные возможности, начиная от разгона целой флотилии кораблей до релятивистский скоростей, до схождение Солнца с главной последовательности, удлиняя тем самым срок его жизни.
Аноним 26/01/23 Чтв 17:46:42 741406 486
>>741404
Справедливости ради материя с отрицательным давлением вполне легитимна и появляется во многих моделях. Но ее проявление требует совсем уж диких условий вроде таких какие были в первые моментах жизни Вселенной или как вариант темной энергии, которая слишком слабая и не допускает способов манипуляцией с ней.
Аноним 26/01/23 Чтв 18:13:32 741410 487
>>741406
Пузырьдебил, таблетки.
Аноним 26/01/23 Чтв 19:44:16 741421 488
Аноним 26/01/23 Чтв 20:28:23 741426 489
>>741366
Это решение уравнений общей теории относительности, дегенерат, он как раз-таки на бумаге и существует. Технически мы его не можем собрать, поскольку еще недостаточно развиты, но принципиально варп-двигатель более чем возможен.
Аноним 26/01/23 Чтв 21:02:26 741436 490
>>741404
>Блять
БляДь.
Заебал, писать правильно научись, потом что-то высирай, долбоёб.
Аноним 26/01/23 Чтв 21:02:50 741437 491
>>741405
Ибатуллин, перелогинься. :)
Аноним 26/01/23 Чтв 21:20:44 741439 492
>>741403
>>741404
Ну если отойти в сторону от маняфантазий мочоных, которые зацыклились на своих формулах и не видят реальности, то вполне возможен.
>Нет способов манипуляцией с гравитацией
Если ты их не видишь, это еще не значит что их нет. Есть, просто их еще не придумали или они не на слуху.
>отрицательная материя
Буэээ.

Кароч, если совсем сложна в гравитационное поле, то ничего страшного, есть друге поля, в том числе еще не открытые. Все поля по своему действию похожи. У всех полей внутри на фундаменатальном уровне одно действие - взаимодействие материи. Один квант материи "ударил" другой и пероизошла передача движения. Это самая база. Так шо не стесняемся, берем для начала электромагнитное поле и задрачиваем его. Делаем самоускоряющиеся ячейки на замкнутых на себя электромагнитных полях. Это даст в пределе скорость света. Для начала хватит полетать по солнечной системе.

Да, варп с ходу сложно освоить, надо сначала просто смочь в безопорное движение, хотя бы с тягой как у ионных движков. Потом прокачивать то что есть и достичь предела для электромагнитных полей - скорость света. Дальше уже можно будет думать о варпе, так как возможности появятся.
Аноним 26/01/23 Чтв 22:17:57 741451 493
image.png 82Кб, 475x290
475x290
>>741439
>отойти в сторону от маняфантазий мочоных, которые зацыклились на своих формулах и не видят реальности, то вполне возможен
Чел, челик, челибас, челидзе, челябинский метеорит... О чем это я? Ах, да. Челюскин, наука так не работает. Ты не можешь выкинуть на хуй формулы и начать работать над чем-то потому что тебе хочется чтобы реальность была такой безо всякой подлежащей базы.
Аноним 26/01/23 Чтв 23:37:18 741458 494
> кококо учёные в будущем разберутся, создадут
Окей. Учёные создадут. А где, сука, тогда корабли пришельцев? Наверняка есть цивилизации, которые уже миллионы лет развиваются. Их моченые уже бы создали Варп Двигатель. И они бы всю Вселенную засрали нахуй с такими возможностями.
Аноним 27/01/23 Птн 00:50:20 741465 495
>>741458
>А где, сука, тогда корабли пришельцев?
Ты будешь смеяться, но вполне возможно, набери воздуха в грудь, ответ на этот вопрос таков: человечество это Предтечи, тобишь мы первая жизнь в галактике =самые развитые на данный момент
Как тебе такой поворот?
Аноним 27/01/23 Птн 00:55:54 741467 496
>>741465
Предтечи это в 99% случаев кринж
Аноним 27/01/23 Птн 01:01:27 741468 497
>>741467
Как раз мы!
Представляешь кто-то найдёт флешку с мемами 2007 года через 1000000 лет, вот потомки обосруться от сего артефакта.
Аноним 27/01/23 Птн 01:50:59 741470 498
image 939Кб, 900x500
900x500
>>741465
Допустим, что каким чудом мы оказались в чудовищно огромной галактике одни такие умнички. Но ведь Варп-Двигатель это такая вундервафля, которая ебёт пространство-время, ебёт тёмную энергию и даже самого аллаха. То есть сюда бы уже прилетели гости из других галактик, если бы Варп-Двигатель существовал. Причём они бы не просто прилетели, они бы начали пожирать звёзды, чтобы заряжать свой магический двигатель. И мы бы это заметили.
Аноним 27/01/23 Птн 02:02:23 741472 499
GalaxiesoftheIn[...].jpg 5942Кб, 4600x2300
4600x2300
Двач. Кто нибудь сталкивался с такой теорией?
Все вселенные, галактики, звезды, двигаются по законам одной большой галактики или что то типо того (галактики, звезды, - это конструктивные элементы) (черные дыры - деструктивные).
Но! В то же время у каждой вселенной, звезды, материи, черной дыры, галактики, есть свои законы движения и. т. п
Это применимо также и в реальной жизни к корпорациям, компаниям, человеку в частности.
Аноним 27/01/23 Птн 02:03:41 741473 500
image.png 487Кб, 1280x720
1280x720
>>741470
Может звезды не надо пожирать, может вообще наоборот межзвездные полеты генерят материю, может это причина барионной асимметрии или темной материи, как тебе такое, Илон Маск?
Аноним 27/01/23 Птн 02:04:15 741474 501
>>741472
Что за теория? Ссылка есть?
Аноним 27/01/23 Птн 02:07:15 741475 502
>>741474
Вот я и спрашиваю, у меня перед сном она как на яву повторяется.
Аноним 27/01/23 Птн 02:18:48 741477 503
qARRNdgv0vM.jpg 114Кб, 500x600
500x600
>>741458
>где, сука, тогда корабли пришельцев?
Аноним 27/01/23 Птн 02:20:23 741478 504
800px-NGC4319IH[...].jpg 138Кб, 800x796
800x796
>>741472
Вот что получилось нарыть в гугле, квазары - ядра галактик, мб существует одно большое ядро которое вращает весь космос, как Ашот трубу?
Аноним 27/01/23 Птн 03:45:34 741482 505
>>741470
>сюда бы уже прилетели гости из других галактик
Да с чего вдруг?
У нас молодой кластер Вселенной, свет то всё равно перемещается с одной скоростью, в соседнем кластере может даже не знают о том что рядом Большой Взрыв произошёл, а думают что там Вселенная кончилась, либо банально Варп-Двигатель не на что нацелить, не будешь же ты в пустоту лететь.
Вот как свет от нашего кластера до них долетит - так и жди гостей. К тому времени мы сами можем уже к ним прилететь, ибо нехер.

>>741478
>мб существует одно большое ядро которое вращает весь космос, как Ашот трубу?
Навряд ли, иначе бы нас скукожило как Бог черепаху и никакая жизнь не была бы возможна.
Скорее просто отдельные кластеры, каждый размером с несколько видимых нами размеров Вселенной, у каждого свой цикл, заканчивающийся ГигачёрнойДырой с Большим Взрывом и началом по новой, которые между собой внешними силами отталкиваются и вращаются вокруг сами себя или же как вода текут куда-то в бесконечность, не смешиваясь. Хотя может БВ как раз результат столкновения ГигачёрныхДыр на последней стадии, как результат резкого превышения массы и дополнительных скоростей. Эдакий естественный ГигаАдронныйКоллайдер.
Аноним 27/01/23 Птн 07:06:31 741483 506
>>741468
Флешка столько не проживёт, к сожалению. До предтечей с неубиваемыми артефактами нам ещё расти и расти.
Аноним 27/01/23 Птн 07:32:32 741484 507
>>741475
На какую яву?
Если ты только что придумал что-то, это не теория, это "я только что придумал что-то". Тогда будь добр как-то формализовать это "что-то", чтобы можно было как-то это "что-то" конструктивно рассмотреть.
Аноним 27/01/23 Птн 08:36:20 741487 508
Аноним 27/01/23 Птн 10:15:16 741488 509
Аноним 27/01/23 Птн 10:32:33 741489 510
Аноним 27/01/23 Птн 10:39:20 741490 511
Screenshot2023-[...].jpg 394Кб, 720x1520
720x1520
вот еще аргумент для доказательства геоцентрической модели мироздания
Аноним 27/01/23 Птн 10:53:23 741491 512
>>741488
>>741489
>.m.
>не может в нормальные ссылки
Как же заебали мобилобляди в моих интернетах. Иди на хуй, не собираюсь читать твои высеры.
Аноним 27/01/23 Птн 11:06:51 741493 513
>>741491
если ты пидор, это еще не значит что все остальные пидоры
Аноним 27/01/23 Птн 12:02:39 741494 514
>>741482
> не будешь же ты в пустоту лететь
Люди же плыли в пустоту и ничего.
Аноним 27/01/23 Птн 12:31:44 741495 515
>>741494
Мы, наверное, не запомнили и не записали тех кто уплыл и не вернулся.
Аноним 27/01/23 Птн 12:55:52 741496 516
>>741494
Люди не плыли в пустоту, они плыли за вполне конкретными вещами.
Аноним 27/01/23 Птн 14:42:47 741505 517
image 265Кб, 500x500
500x500
СКЛОНИТЕСЬ ПЕРЕД ВЕЛИКИМ АТТРАКТОРОМ!
Аноним 27/01/23 Птн 15:00:13 741508 518
>>741483
>Флешка столько не проживёт
Ставлю на то что хранение информации пойдёт по пути Криптона.
Недавно где-то видал что уже в кристаллы кто-то пытался инфу записывать.
Такое в вакууме может и прожить миллионы лет.
Аноним 27/01/23 Птн 15:41:49 741513 519
Plimpton-053-ro[...].jpg 192Кб, 1200x802
1200x802
Аноним 27/01/23 Птн 15:51:23 741514 520
>>741488
>>741489
Ебанутик, спокойствие. Прими таблетки.
Аноним 28/01/23 Суб 12:16:14 741607 521
>>741508
Найдется ли через миллион лет хоть кто-то, кто сможет эти письменя прочесть?
Аноним 28/01/23 Суб 13:29:57 741614 522
>>741467
>Предтечи это в 99% случаев кринж
Уникальность Земли тема вполне серьезная, вполне может быть что такое совпадение условий одно на десяток-другой галактик.
Аноним 28/01/23 Суб 16:17:14 741640 523
>>741607
Даже если и найдётся, даже если кристал и не рассыплется нахуй и инопланетяны его найдут... они ж не заорут во всю глотку - "ебать, на этом кристале палюбас мемасики записаны пойдём его на расшифровщике кристалов прочитывать!" Хуй там, они его просто сразу вплавят куданить в своё сраное оборудование, чтоб его характеристики повысить
Аноним 28/01/23 Суб 22:15:59 741672 524
>>738638 (OP)
Не подскажете, какое атмосферное давление на Венере на высоте 48 километров?
Аноним 28/01/23 Суб 22:37:42 741678 525
image.png 217Кб, 600x362
600x362
>>741672
С подвохом вопрос чувствую я
Аноним 28/01/23 Суб 22:52:47 741686 526
>>741672
Примерно около 1 бара.
Аноним 28/01/23 Суб 22:56:28 741688 527
>>741686
Хм... Тогда на какой приблизительно высоте давление будет в 1,3~1,5 земного?
Аноним 28/01/23 Суб 23:07:07 741693 528
image.png 50Кб, 239x211
239x211
>>741688
Чуть пониже. Я не нашел точной инфы по давлениям, только график где 1 бар на высоте 50 километров и 10 бар на высоте где-то 30 километров.
Аноним 28/01/23 Суб 23:10:55 741694 529
>>741693
Ясно, спасибо. Берем 45 км.
Аноним 28/01/23 Суб 23:31:38 741699 530
image.png 232Кб, 600x422
600x422
>>738638 (OP)
Еще один глупый вопрос. И снова про Венеру.
Допустим, в будущем люди захотят основать на Венере летающую колонию. Зачем - не спрашивайте

Как думаете, кого взять в качестве колонистов: белых или негров? С одной стороны, там повышенный радиационный фон и дофига солнца, с другой - база будет парить в облаках, в относительной тени, а люди будут жить в непрозрачных дирижаблях.
Кого все-таки возьмем для колонизации
Аноним 28/01/23 Суб 23:35:23 741701 531
>>741699
>Кого все-таки возьмем для колонизации
Комсомольцев-добровольцев обладающих необходимыми компетенциями. Не будь расистом.
Аноним 28/01/23 Суб 23:47:32 741702 532
>>741699
Зависит от минимум пары факторов:
1. Того, проводились ли долгосрочные исследования.
Может окажется действительно, что негры сильно лучше переносят радиацию, тогда им приоритет.
2. Если там очень долговременное проживание.
Если вахты по году, то пофиг на то что написано выше, любой огурец подойдет. Если очень долговременные или пожизненные вахты в один конец (что вряд ли), то негры, т.к. дольше прослужат.
Аноним 28/01/23 Суб 23:50:42 741703 533
>>741701
Я не расист.

>>741702
Спасибо. Но, боюсь, тут работа не вахтовая, а именно: в один конец.
Аноним 28/01/23 Суб 23:55:51 741704 534
>>741703
>Я не расист.
А я - да. Но шутка не работает на русском языке, т.к. слова "раса" и "гонка" - разные, и я не могу сказать что я расист потому что не считаю НАСКАР худшей по отношению к Формуле.

>Спасибо. Но, боюсь, тут работа не вахтовая, а именно: в один конец.
Сложно верится. В один конец это таки колонизация, а с Венерой такого без глобальных терраформирующих усилий в это не верится. С Марсом можно поверить, селятся на этой шоколадке, обживаются, строят хабитаты, расширяются, промышленность, ресурсы, гроб, гроб, кладбище пидор, ну ты понел, какое-то подобие автономии.
А на воздушных замках Венеры автономии нет и быть не может, потому не верится что кто-то туда смертников без цели посылать будет.
Аноним 29/01/23 Вск 00:04:02 741705 535
>>741704
>Сложно верится. В один конец это таки колонизация, а с Венерой такого без глобальных терраформирующих усилий в это не верится.

В инопланетную жизнь тоже с трудом верится. Но сколько же про нее книг написано!..

>А на воздушных замках Венеры автономии нет и быть не может

Достигнуть автономии можно, но для этого нужны технологии, на порядок выше наших. Что осуществимо к 23-24 веку.

>потому не верится что кто-то туда смертников без цели посылать будет.

Цель есть. Даже сейчас целей много. А если мы в своей книге сделаем одно маленькое допущение....
Аноним 29/01/23 Вск 00:12:21 741706 536
>>741705
Лады, если история складная и те вещи которые я упомянул объясняются, то конечно можно такое.
Аноним 29/01/23 Вск 00:21:48 741708 537
>>741706
>и те вещи которые я упомянул объясняются
Конечно, за это я взялся едва ли не в первую очередь.
Аноним 29/01/23 Вск 18:52:01 741739 538
>>741705
>Но сколько же про нее книг написано!
Про хогвартс тоже дохуя написано, так что...
Аноним 29/01/23 Вск 22:33:57 741768 539
image.png 47Кб, 220x220
220x220
image.png 44Кб, 259x194
259x194
>>741739
Заставил меня улыбнуться.

>>738638 (OP)
Аноны, я вновь прошу помощи лучших умов Спейсача!

Продолжаю писать свою книгу. Пока события разворачиваются на Земле. Но вскоре действие будет перенесено на другую планету. Проблема в том, что я не знаю, на какую именно...
Сюжет книги таков:

2295 год. Бывшего военного летчика из одной бедной восточной страны нанимает корпорация "Космические Ресурсы" для выполнения боевого задания за пределами Земли.

Дело в том, что одна из внеземных колоний этой корпорации обнаружила на своей планете нечто неизвестной природы. Корпорация затребовала немедленно транспортировать объект на орбиту Земли для изучения, но колонисты показали корпорации средний из 7 пальцев и сказали, что они теперь независимое государство. Мол, устали мы терпеть гнет Земли и прочая, и прочая...

Корпорация идет на переговоры с сепаратистами, понимая, что при удачном раскладе они будут гораздо эффективнее тупой войны. Но "переговоры" оказываются недолгими: после 2 месяцев попыток в диалог, колонисты объявляют компании войну и уничтожают "посольство" корпорации.

Во время боя вертолет нашего ГГ оказывается сбит, а сам он - подобран мимопроходящим транспортом противника. Находясь в плену, из подслушанных разговоров ГГ понимает, что транспорт направляется с целью "зачистки" какого-то сверхсекретного "Объекта-0" (а его подобрали просто "по пути").

ГГ ничего не понимает до тех пор, пока с транспортом его пленителей не происходит авария. Чудом спасшись, он оказывается один на неизвестной територии. Вскоре он понимает, что это и есть "Объект-0". Ему предстоит узнать, какие тайны скрывает это место, почему оно настолько секретно, и от кого его хотели зачищать...


Собственно, вот. И все бы ничего, но я до сих пор не определился, какую планету выбрать: Титан или Венеру .

С одной стороны, Титан хорош. Это практически идеальный кандидат, но есть несколько проблем:
1. Особо ничего ценного на самом Титане нет.
2. Лететь очень далеко и долго.
3. Планета очень известная и "раскрученая" (ее не обошли стороной даже Лем и Азимов).

С другой стороны, есть адская невыносимая Венера, где, тем не менее:
1. Можно добывать дейтерий из атмосферы и плутоний из коры (при помощи очень продвинутых роботов).
2. Лететь совсем недолго.
3. Можно строить воздушные острова-аэростаты (соответственно, однии из них может быть "Объект-0"), целые флотилии таких штук.
4. Планета малопопулярная в научной фантастике.

Я разрываюсь на части. Пожалуйста, помогите неопытному юному графоману сделать правильный, научно обоснованный выбор!
Аноним 29/01/23 Вск 23:03:52 741769 540
>>741768
На Титане обнаружили силикатные формы жизни которым вполне норм в жидких углеводородах.
Аноним 29/01/23 Вск 23:19:38 741770 541
>>741769
Нет, я про полезные ископаемые.
(Что там за "Нечто" обнаружили, я уже придумал.)
Какие такие полезные ископаемые могут быть на Титане, чтоб ими заинтересовалась корпорация?
Или вы предлагаете импорт именно местной "фауны"?
Аноним 29/01/23 Вск 23:40:07 741774 542
>>741770
Косминченская межплонетовая цивилизация полезные ископаемые в гравитационных ямах копать не станет, пока есть астероиды.
Можно придумать какой-нибудь йоба-процесс который эта биота хорошо ебашит. Правда, криожизнь хуета, слишком медл... О, я вроде понел, она на Титане хуйня, зато как в тепло вытащишь, метаболизм на порядки ускоряется и они ебашут дофига энергии или полезного выхлопа, но умирают быстро, потому надо их экспортировать?
Ну такое, я не могу всерьез рассуждать не вникнувшись в идею.
Аноним 30/01/23 Пнд 00:18:27 741780 543
>>741774
Я кое-что придумал для нашего товарища фантаста.
Пусть в его мире богачи живут неограниченно долго: лет 200, 300 и т.д. Единственное, что их может убить - взбунтовавшиеся подчиненные, войны или катаклизмы.
Корпорация строит на Титане масштабные хабитаты, куда позже переселятся богачи с неспокойной Земли. Процесс идет мелденно (лет 50), в нем задействованы куча людей, в основном - наемные рабочие из бывшего СНГ.
Постепенно колония разростается и становится абсолютно самодостаточной. Населяют ее уже под 1КК человек. В какой-то момент они задумываются: а не послать бы нам богачей с Земли на... ? Почти одновременно с этим они находят НЕЧТО во льдах и понимают, что это их шанс.
Они убивают лояльных Земле людей и говорят: "Ну-ну, придите к нам!"
Корпорация понимает, что вести войну с Титанцами невыгодно: логистика не позволяет. Поэтому они пытаются уговорить титанцев к сотрудничеству...
Однако наши гордые "отморозки" не покупаются. В конце концов, они устраивают войну в которой, очевидно, поьедят.
Аноним 30/01/23 Пнд 11:12:24 741798 544
Почему в реале не пользуются орбитальной сборкой для крупных миссий к другим планетам или луне? Почему всегда пытаются построить огромную ракету чтобы за раз пустить? Какие подводные у орбитальной сборки межпланетных аппаратов
Аноним 30/01/23 Пнд 12:00:01 741800 545
>>741798
>Почему в реале не пользуются орбитальной сборкой для крупных миссий
Пользуются, МКС так построили
>к другим планетам или луне
А тому що нема этих миссий
Аноним 30/01/23 Пнд 14:51:29 741827 546
>>741768
> С другой стороны, есть адская невыносимая Венера, где, тем не менее:
> 1. Можно добывать дейтерий из атмосферы
на венере нет водорода, ни в атмосфере, ни на поверхности
> плутоний
его тоже нет, ни на одной планете в сколь либо значимых количествах, весь плутоний на земле выработан на реакторах
Аноним 30/01/23 Пнд 17:22:55 741844 547
>>741827
>на венере нет водорода, ни в атмосфере
Якобы есть.
Аноним 30/01/23 Пнд 17:25:07 741845 548
Аноним 30/01/23 Пнд 17:31:47 741846 549
Screenshot20230[...].png 250Кб, 720x1023
720x1023
Аноним 30/01/23 Пнд 19:15:09 741854 550
>>741845
>.m.
>не может в нормальные ссылки
Как же заебали мобилобляди в моих интернетах. Иди на хуй, не собираюсь читать твои высеры.
Аноним 30/01/23 Пнд 20:05:15 741859 551
>>741854
Молодой человек, вы обосрались.
1. Высеры не мои.
2. Интернеты не ваши.
2. Дейтерия на Венере дохуя Только вот тащить его на Землю экономически совершенно не выгодно, что мы автору-долбоебу не скажем.
Аноним 30/01/23 Пнд 21:01:41 741864 552
Аноним 30/01/23 Пнд 23:07:30 741880 553
>>741864
Что под спойлером?

мимо
Аноним 30/01/23 Пнд 23:27:59 741882 554
>>741880
Наведи курсор и посмотри, ебанько.
Аноним 30/01/23 Пнд 23:51:07 741884 555
Вопрос больше про нашу земляшку, но и про другие планеты тоже было бы интересно почитать.
1. Насколько глубоко под уровнем моря могут встречаться пещеры, полости и каверны? Насколько вероятно буря, например, новую глубокую скважину типа Кольской наткнутся на пещеру или другую полую подземную структуру?
2. Что из себя на физическом уровне представляет мантия планеты? В ней не могут возникать полости?
3. Почему сегодня в других странах не пытаются пробурить новую исследовательскую скважину типа Кольской?
Аноним 31/01/23 Втр 00:06:39 741885 556
>>741884
> Насколько глубоко под уровнем моря могут встречаться пещеры, полости и каверны? Насколько вероятно буря, например, новую глубокую скважину типа Кольской наткнутся на пещеру или другую полую подземную структуру?
Немаленькая. Но что ты называешь полостью? Скажем газовое месторождение - оно полое или как? Это в 5 км под землей встречается.
Мантия - вязкая, под давлением. Как там возникнет полость? Что будет ее поддерживать? Давление газа, как пузырек? Но под высокими давлениями и температурами газы растворяются в веществе мантии, пузырек впитается в стенки и схлопнется.
Постоянно бурят скважины. Всем подавай газ, нефть, уран, соль, и все что добывается выщелачиванием.
Просто больше некоторой глубины уже идет ебаная лава, будешь сильно глубоко бурить - потеряешь бур. Оно тебе надо?
Вот потыкаются, посмотрят, что выбуривается, и часто бросают, ибо не нашли ничего.
Аноним 31/01/23 Втр 00:10:36 741887 557
image.png 616Кб, 970x580
970x580
>>741882
А вот тебе, мошенник!
Аноним 31/01/23 Втр 00:15:43 741888 558
>>741880
Под спойлером то же самое, что и над ним.
Аноним 31/01/23 Втр 00:19:10 741889 559
Вопрос!

По идее...
1. Тритий возникает в верхних слоях атмосферы из азота.
2. При распаде из трития получается гелий-3.
3. Близкая к Солнцу планета с азотом (3 5%) в атмосфере - Венера.
Вывод: на Венере должно быть больше гелия-3, чем на Луне?

Где ваш слуга ошибается?
Аноним 31/01/23 Втр 00:23:24 741890 560
>>741889
Не нашёл публикаций про гелий-3 на Венере.
Про дейтерий-то есть, а про гелий-3 ничего. Правда, про дейтерий небольшая вырезка датированная 1991 годом. Одна.
Такое ощущение что на венеру забили.
Аноним 31/01/23 Втр 00:25:39 741891 561
>>741890
Вот это мне показалось странным... Может, действительно на Венеру все забили? Может, анализов на гелий-3 и не проводилось?
Аноним 31/01/23 Втр 00:37:55 741895 562
>>741889
Возможно, весь гелий-3 сдувает с планеты. С другой стороны, обычного гелия там много и он как-то не улетает.
Аноним 31/01/23 Втр 00:58:36 741897 563
Еще теории будут?
Аноним 31/01/23 Втр 01:52:54 741898 564
Аноним 31/01/23 Втр 07:55:59 741904 565
>>741845
> Доля дейтерия составляет 0,015—0,025, что в 100—150 раз выше, чем земное значение 0,00016
красивые циферки, поигрались с множителями, но даже по самым округлённым в венерианскую сторону значениям - всего на 25% больше чем в атмосфере земляхи(и 0,1% от океана), причем это во всей стобарной атмосфере
на высотах где одна атмосфера не на что смотреть
Аноним 31/01/23 Втр 08:36:22 741907 566
>>741904
>не на что смотреть
У меня возникла теория, что там есть гелий-3 и его больше, чем на Луне...
Аноним 31/01/23 Втр 08:56:03 741911 567
>>741889
ошибся в том что гелий3 на луне копится в камушках на поверхности, механизмов его накопления на поверхности и в атмосферах планет земной группы нет
Аноним 31/01/23 Втр 09:01:09 741912 568
>>741911
>в атмосферах планет земной группы
Чем не подходит описанный механизм? Даже на Земле гелия-3 достаточно, чтоб надуть шарик , а уж на Венере...
Аноним 31/01/23 Втр 10:03:30 741913 569
>>741912
тем что атмосфера не накапливает лёгкие водород/гелий и они улетучиваются в космос куда больше прочих
весь доядерный гелий3 - медленно выходящий из мантии и глубин коры изначальный материал солнечной системы

образования из азота это больше курьёзная статистическая смехуечка чем источник, и даже его продукт скорее всего не останется на планете, а полетит сразу нахуй

вообще на меркурии и безатмосферных камнях вне магнитного поля тоже возможен лунный механизм, но лунный открыт по спектрометрии образцов, без их доставки в хорошем количестве говорить особо не о чем
Аноним 31/01/23 Втр 10:17:39 741915 570
>>741913
Ага, понятно. Спасибо.
(Я так понял, на Венере гелий-3 искать бесполезно.)
Аноним 31/01/23 Втр 13:21:04 741921 571
image.png 353Кб, 768x576
768x576
Поиграл во фрилансер и увидел там черную НЕХ, которая называется "нейтронная звезда". В оригинале она тоже черная была, в ХД версии тоже черная, выглядит как обугленная планета. Почему так, разве НЗ не должны быть яркими? Что могло послужить причиной такой визуальной интерпретации этого небесного тела?

Алсо, там дохуя зеленых звезд, такое может быть?
Аноним 31/01/23 Втр 13:46:18 741927 572
>>741921
>Почему так
Этот вопрос надо задавать разработчикам. Не думаю, что они сидят итт.
>разве НЗ не должны быть яркими?
Конечно должны. В реальности они и есть охуительно яркие. В инфракрасном, оптическом, радио-, гамма- и ультрафиолетовом диапазонах.
>там дохуя зеленых звезд, такое может быть?
Нет, такого цвета нет ни у каких звезд. Если говорить о нейтронных звездах, то, поскольку они охуенно горячие, температура их поверхности намного горячее температуры поверхности обычных звезд, их цвет ближе к фиолетовому.
На скриншоте же изображенное тело больше похоже на черную дыру.
Аноним 31/01/23 Втр 15:53:23 741954 573
>>741927
Мне кажется, что может быть они прочитали что-то про кору нейтронных звезд, ебическую плотность и звездотрясения на них, потому представили их как угольно-черные треснутые шары.
ЧД в игре нет, к сожалению. Или к счастью, в игре шейдеров нет, и без искажений выглядело бы всрато.
У НЗ тоже должны быть искажения, кстати. Только шейдерами это всрато всегда получается, почему-то эти искажения в играх и интерфейс отражают зачастую.
Снова о Венере Аноним 31/01/23 Втр 16:49:33 741967 574
Screenshot20230[...].png 92Кб, 720x402
720x402
Может ли быть на Венере, как на Титане, зона штиля высоко в атмосфере?
Аноним 01/02/23 Срд 00:56:42 742020 575
>>741967
Если бы она была, то нашли бы.
Аноним 01/02/23 Срд 01:22:45 742023 576
Аноним 01/02/23 Срд 12:43:12 742052 577
>>741954
Мб попутали с черными карликами, такие остывшие звезды
Аноним 01/02/23 Срд 12:52:42 742054 578
image.png 443Кб, 640x336
640x336
>>742052
Забавно, что это свежебахнувшая звезда, а события в не настолько отдаленном будущем (где-то 3000 год н.э.) чтобы черные карлики могли существовать.
Мне кажется, что это >>741954 объяснение наиболее простое, просто не знали как выглядит, как я не знал что у ЧД аккреционный диск светлее с одной стороны до Интерстеллара, несмотря на то, что это уже нарисовали в 70-х или 60-х.
Какого хуя до Интерстеллара никто правильно не рисовал ЧД? Почему всегда черные шарики с искажениями, и если с диском, то равномерным? Почему я узнаю о правильном виде из научно-фантастического фильма, когда это известно полвека как, но никто об этом не говорил?
Аноним 04/02/23 Суб 11:49:12 742424 579
>>741000
Нафига,если на околосветовых скоростях это ниочем?
Ответить в тред Ответить в тред

Check this out!

Настройки X
Ответить в тред X
15000
Добавить файл/ctrl-v
Стикеры X
Избранное / Топ тредов